sexta-feira, 11 de abril de 2014

Tem certeza que a tendencia e essa?vc nao deve ler os jornais nem ver jornal na Tvpq é assassinato em cima de assassinato, se a tendencia fosse essa o mundo estaria a cada dia melhor e não cada dia pior,não interessa se essa ideia e pra daqui a 2 mil anos do jeito que tá daqui esse tempo vai ta pior‎ .

Mas o mundo está muito melhor hoje do que já foi há mil, dois mil, três mil, quatro mil anos atrás e assim por diante. O que me faz supor que daqui a mil, dois mil, três mil e assim por diante estará melhor ainda. As liberdades, a prosperidade, o nível cultural, a expectativa de vida, a paz, a ordem a segurança, hoje são muito maiores do que já foram em séculos passados. O problema da criminalidade no Brasil atualmente é circunstancial. Não é a tendência secular.

Se evolução é real porque nenhum chimpanzé evolui até ser como a gente?

Porque não é assim que a evolução acontece. Os Chimpanzés evoluíram de outros macacos que os antecederam, do mesmo modo que nós também. E houve uma espécie foi ancestral tanto da nossa quanto dos Chimpanzés. Quando há uma evolução de uma espécie para outra, a anterior continua a existir. Apenas a descendência do indivíduo que sofreu a mutação é que evoluirá. E a evolução não é algo que aconteça sistematicamente. Pode ser que os Chimpanzés nunca mais evoluam além do que são. Ou que algum Chimpanzé evolua e comece um outro ramo que dará em outra espécie. Mas qual vai ser, não se sabe. A evolução dificilmente se repete. Certamente que não será humana. Da mesma forma que nós poderemos evoluir para outra espécie. Que poderá coexistir conosco e nós seremos para ela como os
Chimpanzés são para nós. Todos os seres vivos atuais são o produto da evolução de seus ancestrais até o primeiro ser vivo. Todos são tão evoluídos quanto nós. Outra coisa é que a especiação só se dá ao longo de milhares de gerações. Nós estamos evoluindo e a maioria dos seres vivos também está. Mas a percepção de que surgiu outra espécie só acontece ao longo de milhões de anos. Exceto para seres cujas gerações sejam muito breves, como as bactérias. Essas estão evoluindo literalmente a olhos vistos.

Por que querer buscar uma verdade? Por que não simplesmente viver essa realidade e aceitar que não somos capazes de obter uma verdade absoluta?

Porque essa realidade é funesta. É uma realidade nociva à felicidade geral. É uma realidade baseada em falsas premissas, em concepções equivocadas. É claro que não se pode, nunca, saber que se é possuidor de uma verdade de modo absoluto. Exatamente por isso é que a busca da verdade tem que ser permanente, pois o que se obtém é uma aproximação assintótica dela, Há que se, sempre, duvidar de que se possua a verdade e envidar esforços para testá-la e buscá-la, cada vez mais se aproximando dela, ou mesmo, alcançando-a, sem nunca se estar certo de o ter feito.

Professor, como lidar com a angústia gerada por não entender o Universo?

Dedicando-se a entendê-lo, estudando cosmologia. É bom, também, estudar física de partículas e o aspecto filosófico do assunto. Enquanto você está estudando, mesmo que ainda não chegue ao entendimento, a angústia acaba, pois você está agindo para que a razão de sua angústia seja sanada.

Professor,gostaria de saber em quais livros o senhor aprendeu sobre evolução.Além disso,qual seria seu autor favorito no quesito biologia ?‎

O que é Evolução - Ernst Mayr - Rocco
O LIvro de Ouro da Evolução - Carl Zimmer - Ediouro
Evolução - Mark Ricley - Artmed
Análise Evolutiva - Freeman & Herron - Artmed
Darwin e os Grandes Enigmas da Vida - Stephen Jay Gould - Martins Fontes
À Beira d'Água - Carl Zimmer - Zahar
A Escalada do Monte Improvável - Richard Dawkins - Cia das Letras
O Relojoeiro Cego - Richard Dawkins - Cia das Letras
O Maior Espetáculo da Terra - Richard Dawkins - Cia das Letras
A Grande História da Evolução - Richard Dawkins - Cia das Letras
Isto é Biologia - Ernest Mayr - Cia das Letras
Biologia, Ciência Única - Ernest Mayr - Cia das Letras
Biologia - Campbell & Reece - Artmed

Ernesto, o sr não acha que o analfabetismo político é o maior obstáculo para o funcionamento do Regime Democrático?

Acho que é um grande obstáculo sim, mas não o maior. O maior é a falta de ética dos políticos e dos eleitores. Eleitores ignorantes, mas éticos não votam em candidatos corruptos. É claro que uma das formas de se elevar o nível ético da população é elevar o nível de conhecimento, especialmente político. Mas há muita gente de alta cultura que não é ética. Acho que conduta não ética deveria ser razão suficiente para se cassar os direitos políticos e outros das pessoas. Não estou falando de conduta criminosa nem imoral. Estou falando de conduta não ética.

Como você chegou a conclusão que existe vida fora da Terra?‎

Eu não cheguei a essa conclusão. Eu suponho que seja possível haver, uma vez que as condições, mesmo muitíssimo raras, que se apresentam na Terra para o surgimento da vida, podem se dar em outros lugares. Já foi calculado que um em cada 300 bilhões de planetas pode exibir essas condições. Isso daria, mais ou menos, três Terras por galáxia. Isso quanto a vida inteligente. Vida apenas, do tipo bacteriana, isto é, procariota, pode ser bem mais abundante, talvez entre um centésimo e um décimo dos planetas. Já a eucariota, mesmo microbiana, deve ser muito mais rara. A evolução de célula procariota para eucariota, mesmo em organismos unicelulares é um passo muito maior do que a evolução de protozoários para metazoários, por exemplo.

Professor, não criticando seu ateísmo, mas você não acha que é muito complexo todas as formas de vida aqui terra, onde bem dizer tudo se encaixa perfeitamente, prosperando a vida, uma inclinação maior no eixo na terra, uma atmosfera menor, já faria de nós seres inexistentes..pense nas possibilidades

Claro que é muito complexo. E muito improvável. Mesmo assim, é possível e é a única explicação plausível. A suposição da interveniência de alguma entidade extrínseca ao Universo no desenrolar dos acontecimentos que levaram ao seu surgimento, à sua evolução, ao surgimento da vida e de vida inteligente, é muito mais improvável do que a de que tudo tenha acontecido por acaso. Porque essa entidade requer a consideração de uma categoria de realidades totalmente espúria e gratuita. Não há indicação nenhuma da existência de tal tipo de realidade. As ocorrências naturais que levam a tudo isso, mesmo que improváveis, são perfeitamente possíveis, dentro do comportamento normal da natureza. E baixa probabilidade, mesmo que muitíssimo baixa, não significa impossibilidade. Mas essa de uma realidade sobrenatural excede qualquer nível de razoabilidade.

Tudo o que é observado é afetado pelo observador?

O que afeta um sistema é o fato dele enviar mensageiros que informem o seu estado, como fótons de luz. É claro que para ser observado, é preciso que um observador capte esses mensageiros. Mas o observador pode ser um aparato inconsciente. Se o sistema enviar mensageiros e não houver nada que os capte para registrar alguma observação, mesmo assim o seu estado ficará modificado. Não é o observador que altera o estado do sistema observado. É o fato do sistema observado comunicar o seu estado, mesmo que nada capte essa comunicação. Então algo, mesmo que não esteja sendo observado, mas esteja enviando mensagens sobre o seu estado, estará continuamente mudando de estado. Quando se coloca um termômetro para medir a temperatura de uma pessoa, a pessoa transfere calor ao termômetro para que ele indique a temperatura. Então sua temperatura já não é a mesma que tinha antes de colocar o termômetro (se nada mais provocar alteração). Mesmo que nenhum ser consciente se inteire da indicação do termômetro, a temperatura do sistema que o termômetro está indicando já mudou.

Que piensas de la tauromaquia?

Um absurdo sem nenhuma justificativa. Uma crueldade que mancha a espécie humana. Inominável. Do mesmo modo que a caça à raposa e qualquer esporte que se baseie em crueldade para com os animais. E crueldade para com as pessoas humanas também, como o boxe.

Todos temos ilusões. Talvez a sua seja a de que o ser humano possa desenvolver um sistema de governo perfeito no futuro. Eu como não sou ateu, acredito que Deus está dando e sempre deu chance pra que o homem crie seus sistemas ("A democracia é o menos pior deles") Acredito que a tal Nova Ordem Mund‎

O que se pode dizer é que muitos de nós têm ilusões, mas não se pode garantir que todos a tenham. E nem que ter ilusões seja uma necessidade psíquica. Minha suposição de que um mundo ateu e anarquista seja possível acontecer no futuro não é uma ilusão. É um desejo completamente alicerçado em indicações históricas. Pode-se ver que a marcha da humanidade, nos últimos milênios, tem sido na direção de cada vez mais liberdade e cada vez mais ceticismo. Pode-se, pois, prever que em alguns milênios a liberdade vá ser total e as crenças infundadas abolidas. Mas isso pode ser reduzido a alguns séculos apenas, por meio de um trabalho constante e abrangente de convencimento e conscientização das pessoas. É isso que venho me propondo a fazer e fazendo ao longo das últimas décadas. Uma das melhores formas disso é por meio da educação científica, histórica e religiosa. Por isso é que sempre me bati pelo ensino de Física Moderna (quântica, relatividade, cosmologia) no Ensino Médio, de Religiões (e não "religião"), de Filosofia, de Sociologia, de Política, de Economia, de Psicologia, de Neurociências, de Evolução. Foi estudando isso tudo por minha conta que me tornei ateu e anarquista. Você não completou o que ia dizer sobre a Nova Ordem Mundial.

"O inteligível é sensível somente ao inteligível?" Fizeram.me essa pergunta mas eu não soube responder. Sabe responder?‎

Essa é uma pergunta mal formulada. Inteligível é o que seja capaz de ser entendido. E entender não é apenas perceber. Entender envolve o conhecimento do que seja a coisa, bem como de suas razões para ser o que é e como é, do modo como acontece, se for um fenômeno e não um ser. Envolve também a compreensão de seu relacionamento com o resto do mundo, suas consequências, suas aplicações, suas limitações e tudo o mais que se possa saber a respeito. Algo inteligível é, pois, o que permite que o intelecto apreenda essas informações a respeito dele. Mas é preciso, por outro lado, que o intelecto tenha a capacidade de estabelecer essas relações. Então, nem tudo que seja inteligível é totalmente assimilado por qualquer mente. Há que ela tenha uma inteligência capaz de tal proeza. Acho que a frase seria melhor enunciada como: A intelecção de algo inteligível só se dá a uma mente de inteligência suficiente para tal. Mas, ele própria, a mente, não precisa ser inteligível. Mas é, mesmo que ainda não seja completamente entendida, não há mistério nenhum na mente que vede a sua intelecção pela ciência, o que se dará, certamente, decorrido tempo suficiente para a compreensão de todos os seus fenômenos.

Considerando que acreditar num deus pessoal seja mera ilusão, e que todos os seres humanos necessitam de ilusões pra viver, qual seria a sua ilusão, visto que é ateu?

Não é verdade que todos os seres humanos precisem de ilusões para viver. Não precisam não. É muito melhor viver sem ilusões. Mas pode-se viver para a construção de projetos para o futuro. Só que eles não são ilusões. No meu caso em vivo para acabar com a ignorância do mundo. E faço de tudo para isso. Também luto pela divulgação do ateísmo e do anarquismo, que considero que sejam as concepções corretas de vida e de sociedade. Sei que não conseguirei obter o resultado que almejo em minha vida. Isso, contudo, não me esmorece o esforço de trabalhar em prol desses ideais. Ideal não é ilusão. É uma visão de futuro, com grandes probabilidades de vir a acontecer.

Professor, final do ano quero prestar vestibular pra eng. química na UFPR. Vou fazer esse ano de Física na UTFPR, e estou fazendo cursinho. Mas me surgiu a oportunidade de um curso técnico de Química. O senhor acha que devo tentar fazer os 3 ou largar o cursinho e fazer técnico e faculdade? Qual vai me preparar melhor para o vestibular?

Acho que seja preferível você fazer o técnico de Química, pois isso já te dá uma profissão. Enquanto isso, nas horas vagas, vá estudando sozinha para o vestibular e larga o cursinho. Cursinho só vale para te dizer o que tem que estudar. O importante é o estudo pessoal. Você pode saber o que vai cair pelo edital do vestibular e ir estudando. Se tiver dúvida, pergunte aos professores do curso técnico.

Ainda sobre crenças, o senhor acredita que todas as pessoas são capazes de encarar a realidade do mundo sem a ilusão de um deus pessoal?

Claro que são. Basta que sejam devidamente instruídas. Especialmente para que tenham um código ético de conduta não baseado em prêmio e nem em castigos além da vida, mas na responsabilidade pela manutenção de uma sociedade harmônica, fraterna, pacífica, justa, ordeira, aprazível, próspera e feliz. para todos e não só para alguns. Isso só pode acontecer se todos forem honestos, prestativos, solidários, de conduta ilibada. A introjeção de uma ética humanista na cosmovisão das pessoas é dever da escola e ter escolaridade é dever do estado e da população propiciar a todos. Quanto às consolações e a esperança que as religiões propiciam, que são completamente ilusórias, elas podem, perfeitamente, serem concedidas pela filosofia e pela ciência, desde que isso seja colocado ao alcance da toda a população. E, mais uma vez, isso é papel da escola. E é papel do governo, enquanto existir, propiciar assistência médica de qualidade a todos, de modo que a confiança no poder da medicina aja em substituição à confiança no poder de milagres, que não existem. Para tal seria preciso que a medicina pública, ou seja, o SUS, fosse o ÚNICO recurso médico disponível para todos, independentemente de seu poder aquisitivo. Ou seja, que não existisse medicina privada. Como também não existisse educação privada. Até que a anarquia abolisse o estado e o governo e a educação e a medicina não fossem estatais, mas fossem totalmente gratuitas para todos, já que não existiria dinheiro.

Se o ar é matéria, então o vento que é o que o empurra é considerado energia?‎

Não. O vento não "é" energia, pois energia não é uma coisa. O vento é ar em movimento que "possui" energia cinética. Energia é um atributo, uma propriedade das coisas. Não existe "energia". O que existe são coisas que possuem energia.

Prof,o senhor tem alguma experiência com o vestibular do ITA? poderia dar alguma dica para quem se prepara para ele?‎

Não conheço particularmente o vestibular do ITA. Mas eu sei que para se passar em qualquer vestibular, seja onde for, basta que se saiba a matéria. Não precisa se preocupar com o que cai ou não cai e com o tipo de questão que se pede. Basta saber muito bem a matéria. Estude para saber. E saber muito mesmo, em profundidade e abrangência superlativas. Então, seja o que for que cair, se saberá responder. Acho isso tão simples que não vejo razão nenhuma para a existência de cursinhos vestibulares. Meta a cara na matéria e estude muito, muito mesmo. Busque se aprofundar além do que os livros passam. Peque livros de nível superior e estude por eles. Não fique preocupado por estar estudando o que não vai cair. Queira saber tudo, o máximo que conseguir. Claro que compensa pegar questões das universidades mais puxadas e resolvê-las para testar como está o seu nível de conhecimento e de habilidade. Pois não basta saber, é preciso saber fazer as questões. É como aprender a nadar. Não se consegue se não se entrar na água. Mas primeiramente tem que saber a teoria, para depois fazer aplicações. Só que as aplicações também dependem de outros conhecimentos, não específicos do que se está estudando. Como ser bom em interpretação de texto e ter um bom vocabulário. E ter um grande traquejo em matemática, até mesmo do ensino fundamental. Muitos erram questões porque não sabem fazer as contas. Isso é algo que depende de toda a vida estudantil pregressa do aluno. Para conseguir entrar para o ITA é preciso que se tenha sido, na escola toda, um aluno muito bom em desempenho acadêmico. Isso não se adquire no último ano de estudo.

Quais as suas considerações sobre essa regra que nossa sociedade tem de que não devemos questionar e criticar as crenças alheias?

Não sei dessa regra não. Claro que se pode questionar tudo, desde que de modo respeitoso. Religião se discute sim e é muito importante se discutir. Para que as pessoas abram os olhos e não vivam iludidas com crenças infundadas. Inclusive, para mim, é um dever de caridade fazer isso.

O que acha dessa frase Ernesto? ''Um exemplo da existência de Deus é a complexidade do Corpo Humano, afinal, como simples poeiras poderiam nos criar com capacidade de pensar conhecer e imaginar?'

Isso não é argumento nenhum para a existência de Deus ou de algum projeto. A evolução pelas mutações fortuitas e pela seleção natural dá perfeitamente conta de explicar o surgimento de níveis complexos de estruturas neurológicas, capazes de possibilitar a inteligência humana, bem como inteligências mais avançadas ainda que, muito provavelmente, irão surgir ao longo do prosseguimento do processo evolutivo no futuro. Como pode acontecer? Pelo acaso e pelas coincidência. Que, aliás, é o fator mais relevante para tudo no mundo, inclusive para o desenrolar de nossa vida pessoal.

O que o Sr. acha das pessoas que guiam as vidas (ou buscam algum tipo de "esclarecimento sobre o futuro/personalidades") na astrologia?

Que estão totalmente iludidas sobre a possibilidade de ser algo confiável. Trata-se de um total despropósito. Não tem o menor fundamento e não funciona. É preciso esclarecer essas pessoas de seu engano e tirá-las da ignorância.

Qual sua opinião a respeito da cela especial para os que tem curso superior?

Completamente despropositada. Crime é crime e todos têm que ser punidos pelo crime, de modo completamente igual, independentemente do grau de instrução ou do poder econômico que possua. Mas, pelo que me consta, isso já foi abolido. Outras coisas que são completamente injustas são o instituto da fiança e do sursis. Ferem completamente o princípio de igualdade perante a lei.

Por que a paixão é algo que não se escolhe? Quero dizer, por que não podemos nos interessar por alguém que nossa mente quer?‎

Porque a paixão é inconsciente e não é lógica, enquanto a mente consciente é lógica e pretende escolher uma pessoa para se amar com base em critérios racionais. Mas o sentimento não é racional e é bom que não seja mesmo. A força do inconsciente, do instinto, da intuição e dos afetos é muito maior do que a força da razão. O melhor que se deve fazer é buscar justificativas racionais para a escolha inconsciente. Mas é preciso que se tenha uma concepção de mundo e de vida que concilie o instinto com a razão. Essa é a concepção libertária e anarquista de vida, que não regula o comportamento pelas amarras das convenções sociais ou religiosas. Que aceita os sentimentos profundos e espontâneos como manifestações legítimas dos anseios mentais e corporais. Que não rotula as paixões como convenientes ou inconvenientes, não condenando nenhuma, pelo contrário, aceitando-as e vivendo-as em plenitude. Sem que sejam ocultas, mas inteiramente abertas, consentidas e divulgadas. Para tal é preciso, também, romper qualquer associação entre os sentimentos amorosos e as relações econômicas. Isso é um grave entrave para a felicidade.

Professor, te sigo a pouco tempo e não sei se você já respondeu essa pergunta. Qual sua posição política e porque?

Sou um anarco-comunista pacifista. Isto é, sou a favor da inexistência de estados e governos, bem como do dinheiro e da propriedade. E de uma forma comunitária e colaborativa de condução da economia, a "economia de doação". Isso tudo alcançado de um modo pacífico e ordeiro, sem nenhuma revolução, mas por meio de uma evolução da civilização com a abolição total da pobreza, das desigualdades de oportunidades, da ignorância, dos crimes e demais mazelas sociais. Mas tenho consciência que tal estado só conseguirá ser alcançado dentro de várias dezenas de gerações, isto é, várias centenas ou alguns milhares de anos. Por enquanto, considero que a combinação político-econômica aceitável é a social-democracia, um sistema hibrido de política democrática com economia capitalista controlada. Sou contra o socialismo de estado, que foi impropriamente chamado de comunismo na União Soviética, bem como na China, em Cuba, na Coréia do Norte e no leste europeu, antes da queda do Muro de Berlin. Sou um libertário, mas considero que o anarco-capitalismo seja nefasto para a sociedade como um todo. O melhor caminho para o anarco-comunismo é a pulverização do capitalismo, que transformará todo trabalhador assalariado em sócio, ou seja, em capitalista. Enquanto no capitalismo temos patrões e empregados e no socialismo só temos empregados de um único patrão, nesse modelo só temos patrões, ninguém sendo empregado e nem assalariado. Todos tirando seus rendimentos dos lucros das empresas em que trabalham. Isso é que é muito mais justo e que levará à abolição da propriedade e do dinheiro por falta de necessidade de existir. Do mesmo modo que as leis, a polícia, os exércitos, as fronteiras, os juízes, os advogados, os bancários e banqueiros, os contadores, os políticos e muitas dessas atividades. Até que se chegue á situação de que nada seja de ninguém e tudo seja de todos. Inclusive maridos e mulheres, filhos e pais.

Qual sua opinião sobre os projetos para educação de superdotados? Vi na Wikipédia que existe um projeto em Lavras, MG. Como você foi professor por vários anos, pergunto: Alguma vez teve algum aluno muito mais inteligente do que os outros da mesma turma? Se sim, como fazia para lidar com a situação?‎

Infelizmente, com poucas exceções, como essa de Lavras, não temos apoio para superdotados nas escolas brasileiras, quer públicas, quer privadas, como se tem para especiais em sentido oposto. É uma pena, pois essas pessoas podem dar uma contribuição muito valiosa para a sociedade. Além de, apoiadas, conseguirem muito mais realização pessoal na vida. Já tive alunos assim e a única coisa que me restava fazer é atender fora de classe em conversas, indicando livros e incentivando a que fossem além do que era dado na escola. Mas a escola não permite, por exemplo, que eles vençam as etapas em menor tempo e cheguem ao terceiro grau com 15 anos ou menos. Tenho uma amiga do Facebook que se dedica a isso. Posso te apresentar a ela lá, se você me mandar uma mensagem.

Eu adoro ler, mas há pessoas próximas a mim que não. Elas me perguntam o porquê da leitura ser uma coisa boa na minha vida, e eu nunca sei o que responder.. O senhor poderia me dizer bons argumentos para eu poder argumentar com elas?

Pessoalmente eu adoro ler porque me dá muito prazer. Acho uma delícia. Muito melhor do que ver televisão. Depois porque a leitura me enriquece culturalmente e isso, para mim, é muito valioso. Não por ter alguma utilidade prática ou econômica, mas por me fazer uma pessoa mais completa e colaborar para meu crescimento em virtude e sabedoria. Mais cultura também me municia de mais assunto para conversar com as pessoas e mais saberes para ensinar aos outros. Finalmente porque me propicia desafios mentais na interpretação do texto, no entendimento dos argumentos, o que aumenta minha inteligência.

Se pudéssemos comprovar que os fósseis dos supostos hominídeos são, realmente, hominídeos e não humanos com alguma deficiência - como os criacionistas alegam - acabaríamos de vez com os argumentos teístas, mesmo aqueles mais volúveis.A pergunta é: Há alguma evidência cabal para crermos nisso?‎

Sim. A idade dos fósseis dos hominídeos, muito maior do que a de todos os fósseis humanos.

Professor, o que tem a dizer sobre as evidências que foram captadas pelo telescópio bicep2, no Polo Sul? Será que agora está muito próximo da comprovação da teoria do big bang?‎

Essa descoberta é uma comprovação do período inflacionário que se deu logo após o big bang. A existência do big bang já é comprovada há muito tempo, pela expansão cósmica e pela radiação de fundo do universo. O período inflacionário, indiretamente, já era aceito como verdadeiro também. Mas poderia ser que outras razões explicassem a homogeneidade e a planicidade do Universo. As medidas agora detectadas confirmam a inflação.

Você concorda que "Amor, quando é amor, termina em barraco. Se termina em silêncio, já não era mais nada"? Exponha sua opinião e experiência, caso tenha e queira.‎

Discordo. Amor quando é amor não termina. O que pode terminar é a relação. Mas o amor não. Mesmo que se venha a amar outra pessoa, não se deixa de amar o amor que se tinha. Amor não precisa ser unívoco. Pode-se amar, sincera e profundamente, a mais de uma pessoa simultaneamente.

Existe limite para a tecnologia?

Sim. Não é possível se construir nenhum artefato que funcione contrariando as leis da natureza. Como um moto-perpétuo. Fora isso, é só uma questão de conhecimento, capacidade técnica e disponibilidade de recursos. Esses fatores limitantes, contudo, não são imperativos, são circunstanciais.

Será que o Brasil é realmente uma democracia?. A população não participa diretamente das decisões do estado e os políticos eleitos representam o interesse de uma elite capitalista. Somos iludidos a crer que esta é uma democracia, mas o povo não possui poder político.‎

É sim. Nenhuma democracia, atualmente, é direta. Todas são representativas. Não há condições do povo resolver, diretamente, as questões políticas. O fato de que os políticos representem interesses dos detentores do capital é culpa dos eleitores e dos candidatos que pretendem defender os interesses do povo que não conseguem convencê-lo de votar neles. Isso é um trabalho que tem que ser feito o tempo todo, no intervalo entre as eleições, convencendo as pessoas a não votar em candidatos que só querem saber da política para benefício próprio. Tirando a anarquia não há regime político melhor do que a democracia. Mas a anarquia requer um nível muito requintado de civilização para existir, o que não se tem, por enquanto.

Por que absurdo?

Porque um Deus de bondade não exigiria o sacrifício de seu próprio filho em dores atrozes para se sentir aplacado de sua ira pelo pecado original. Simplesmente perdoaria. E porque não é justo que ele castigasse a humanidade toda pelo pecado só de Adão e Eva. É claro que tudo isso é, meramente, uma lenda, pois Adão e Eva não existiram, nem Jesus era Deus nenhum. Que, inclusive, não existe.

Acha que poderá existir outros seres no universo para além de nós?

Mas existem. A Terra tem milhões de outras espécies vivas não humanas. Acho que você está se referindo a outros seres inteligentes em outros planetas. Entre no Google e digite:
inurl:wolfedler extraterrestre

Professor, você é licenciado também, certo? Suas aulas da parte de pedagogia lhes foram úteis pra ser um professor melhor? Estou tendo aulas na área, mas tenho achados as aulas muito "senso comum", e que elas pouco me acrescentam nesse sentido.

Quando estudei, considerei bem proveitosas as aulas de Psicologia Geral e Psicologia da Aprendizagem. Já a Estrutura e Funcionamento do Ensino, para mim, não precisava ser uma disciplina. Isso é coisa que se aprende com a prática, à medida que se necessita. E a Didática Geral, no caso, não foi muito bem dada. Mas isso não significa que não seja importante. Um bom aluno é capaz de dispensar o professor e aprender sozinho, desde que não se limite a um livro só. O "senso comum", apenas não é suficiente para orientar o professor em seus procedimentos didáticos. Pesquisas já foram feitas sobre a melhor maneira de se encaminhar o processo ensino-aprendizagem e um professor tem que saber fazer uso do que foi descoberto a respeito. A Didática Especial, no meu caso, foi dispensada, pois eu já lecionava e, então, minha experiência docente contou para as horas da disciplina. Não acho que isso seja uma boa coisa. Depois eu fui professor de Didática Especial de Física e não aceitei esse negócio. Eu dava aulas mesmo, sobre como abordar cada assunto para que fosse compreendido pelos alunos. Uma coisa importante, que não é vista no curso, é treinar a retórica, ou o traquejo para falar em público, bem como a dialética, ou a arte da argumentação (não a dialética hegeliana ou marxista), a lógica e a gramática, para saber escrever corretamente. Isso é importantíssimo para um professor de qualquer disciplina.

Professor por que dizem que não compensa ser tecnólogo e por que alguns pessoas e empresas não consideram como graduação de nível superior?

Por um preconceito elitista próprio do modo de ser brasileiro. Um tecnólogo é capaz de atuar perfeitamente bem em um nível intermediário entre um técnico e um engenheiro. E há muitas funções que engenheiros exercem que não requerem um engenheiro para tal.

Que dica o senhor daria pra quem tá começando a ler livros? Digo no sentido de quem não tem interesse pra ler,mas gostaria de ter. Qual dica o senhor poderia dar pra poder dar mais estimulo a leitura?‎

Comece com esses livros de literatura fantástica que estão na moda, como Harry Potter, Crônicas de Nárnia, Senhor dos Anéis e similares. Bem como policiais da Agatha Christie e do Sherlock Holmes. Ou, ainda, Ficção Científica, como os livros do Asimov e outros.

Professor, recomenda algum livro para estudar especificamente para a OBF?‎

Pegue as provas das olimpíadas anteriores e estude por qualquer bom livro do ensino médio para saber a teoria capaz de resolvê-las. Recomendo os do Ramalho, os do Newton-Helou-Doca e os da Beatriz Alvarenga. Mas, para aprender Física mesmo, boa é a coleção do PSSC, já esgotada. Mas isso é para se estudar ao longo dos anos. Todavia não aprecio muito esse negócio de olimpíadas e qualquer tipo de disputa. Acho muito esquisito isso de alguém ter que vencer alguém. Acho que se precisa vencer são as próprias limitações e ajudar aos outros a vencer as deles também. Isto é, que todos queiram que todos sejam tão bons quanto se é e não se deseje ser melhor do que ninguém.

Você é umas das 9 entre 10 pessoas que eu conheço que são a favor da volta dos militares ao poder?

De modo nenhum. Sou inteiramente contra. Mesmo não estando satisfeito com o atual governo sou um democrata convicto. Não acho que uma ditadura militar seja melhor do que a democracia, por mais problemas que ela possa apresentar. Qualquer ditadura, seja de direita seja de esquerda é péssima. O que temos que fazer é eleger políticos decentes e competentes de modo que essa corja de aproveitadores seja defenestrada pelo voto para o resto de suas vidas. E que se abram processos para julgar e punir exemplarmente toda corrupção seja por parte de quem quer que tenha sido feita. Isso inclui políticos de todos os partidos, empresários, juízes, promotores, policiais, funcionários públicos, sacerdotes e quem quer que seja que faça corrupção.

Sobre a sua resposta sobre o homem ter ido a lua ou não, isso não está relacionado a um aspecto ideológico? Pois tal fato aconteceu durante a Guerra Fria, em meio a corrida espacial. Sem contar que o fato de colocar a bandeira dos E.U.A na lua foi um ato político...

Minha resposta sobre a ida do homem à lua é, simplesmente, factual. Isto é, é fato de que isso ocorreu, independentemente das motivações que levaram a que tal ocorrência se desse.

Professor, o que o senhor acha que aconteceria caso misturássemos alma e coração?‎

Depende do que você está entendendo, metaforicamente, por alma e por coração. Realisticamente, nem alma existe nem coração responde por sentimento ou emoção. Tudo se dá no cérebro. Mas, metaforicamente, coração significa afetos e alma elevação espiritual. Ora, elevação espiritual envolve sentimento e intelecto, de modo que já inclui o que o coração significa. Alma, também pode significar essência, isto é, o que há em cada coisa que a faz se o que é. Unir alma e coração seria, pois, colocar o afeto no que cada coisa tem de essencial. Apaixonar-se pelos seres pelo que eles são em si mesmos, e não pelo que aparentam ser. Isso é, exatamente, o que deve ser feito. Note que paixão pode ser amorosa ou, pelo contrário, odiosa. Em suma, deve-se, também, dedicar ódio e desprezo ao que seja ignóbil, abjeto, repugnante. Ou seja, à maldade.

Por que há pessoas, no meio universitário inclusive, que não acreditam que o homem foi à Lua ? Por que elas adotam um ceticismo em relação à ciência, mas não à pseudociência ?‎

Porque ignoram o desenvolvimento científico. Ou seja, são ignorantes. A crença na pseudociência também atende a um desejo de ser capaz de alterar o desenrolar dos fatos naturais à revelia das próprias leis da natureza e conseguir fazer alguma "mágica". A ciência mostra que mágica (e milagres) são impossíveis. Então elas desdenham a ciência e se aferram à possibilidade de mágicas que a pseudociência e as crenças sobrenaturais ensejam. Infelizmente, em vão.

O que te faz se submeter a questionamentos no ask? Isso alimenta seu ego?

O desejo de difundir o conhecimento e espancar a ignorância. Esse é o significado que atribuí à minha própria vida. Me sinto realizado como ser humano em minha atividade docente e o que faço aqui também é uma docência. Com a vantagem de não ser associada a nenhuma remuneração financeira.

Por que física é tão difícil?‎

Mas não é. Tudo só é difícil se você não entende. Acontece que o ensino de Ciências, desde o Fundamental, não é conduzido de modo a provocar nos estudantes um deslumbramento pelo funcionamento da natureza que os leve a desejar compreendê-lo. Além disso, a Física faz muito uso da Matemática para construir seus modelos descritivos desse comportamento. E a Matemática também costuma ser mal ensinada no Nível Fundamental, fazendo muitos estudantes a detestarem. Como o conhecimento de Matemática e Ciências é cumulativo, qualquer falha em algum elo da cadeia acarreta a incompreensão dos elos seguintes e isso desestimula o estudo. Não é fácil recuperar o que ficou perdido para trás, mas não é impossível, desde que se tenha grande interesse em aprender o assunto. Isso tem que ser despertado pelos professores que precisam ser, eles mesmos, fascinados pelo que lecionam, de modo a contagiar a turma.

Professor, como a magnetosfera protege a Terra da radiação solar?‎

Ela desvia a trajetória dos prótons de alta energia emitidos pelo Sol devido à força magnética que exerce sobre eles, fazendo com que contornem a Terra ao invés de caírem nela. Esses prótons, se atingissem a alta atmosfera, colidindo com os átomos do ar, desencadeariam uma cascata de partículas radioativas que seriam prejudiciais à vida na superfície.

Por que o mal existe?‎

Mal pode ser entendido como toda ação que provoque dor, sofrimento, tristeza, prejuízo, dano a algum ser. O mal pode ser de duas origens. Ou é uma ação da natureza ou é uma ação provocada por um agente intencional. Se bem que um agente intencional pode fazer um mal sem a intenção de fazê-lo. O mal feito sem intenção de assim o ser existe porque os acontecimentos do mundo não são todos previstos e planejados, grande parte deles se dando por razões fortuitas, coincidências e acasos. Então pode ocorrer que algum deles acarrete uma consequência maléfica. Quem considera que tudo o que acontece no mundo é porque Deus quis, então o malvado, no caso. é Deus. O mal deliberadamente executado só o é se o for por um ser inteligente e consciente. Minerais e vegetais não o são. E nem todos os animais. Só os animais superiores, em especial o homem, dos que conhecemos, é capaz de executar uma ação deliberadamente malévola. Eles são capazes disso porque sua estrutura mental possui uma liberdade de decisão na execução dos atos, de modo que podem resolver se fazem o que seja benéfico ou maléfico em relação ao objeto de sua ação. Isso só será assim caracterizado se acontecer de modo consciente e livre. Uma ação pode ser maléfica do ponto de vista de quem a sofre mas não poder ser imputada como uma ação má do ponto de vista de quem a exerce, se for executada sem a consciência de que assim esteja sendo ou se executada sob coação incontornável. A existência do mal é um dos argumentos a favor da concepção de que algum deus, se existir, não pode ser bom. Esse é o argumento atribuído a Epicuro. Para que um ser seja denominado Deus, ele tem que ter poder de fazer o que queira, à revelia das leis da natureza. Se ele tem esse poder e não impede a existência do mal, então não é bom. Outra alternativa, mais plausível, é de que não exista tal tipo de ser.

Existe "tarde demais" para se começar a estudar? Quero convencer minha mãe a fazer faculdade de Direito, mas ela se acha "velha demais" com 43 anos.

De modo nenhum. Pode começar a estudar com qualquer idade. Até com mais de 80 anos, se estiver com vigor. 43 anos é nova ainda. Velha é a avó dela. A terceira idade só começa a partir de 65 anos.

É possível afirmar agora que o Big Bang é um FATO 100% comprovado cientificamente? Ou ainda restam dúvidas?

Nada em ciência é totalmente comprovado. Tudo está em aberto e é sujeito a correções e revisões. Aliás, esse é o maior mérito da ciência: nunca ser um conhecimento definitivo. Não existe certeza em ciência. Há quem conteste o Big Bang e apresente argumentos para sustentar sua contestação. Alguns são simplórios, apresentados por aventureiros que acham que, sem ser cientistas, descobriram que os cientistas estão errados. Mas há, também, contestações bem abalizadas, formuladas por cientistas competentes. Todavia, para serem consideradas válidas e substituírem o modelo aceito, é preciso que a maior parte da comunidade da área se renda às argumentações apresentadas e mude o paradigma. Por enquanto, em cosmologia, o paradigma é o Big Bang. Ou seja, é a explicação com maiores argumentos e maior número de cientistas a defendê-la.

Oi, sou professor de física aqui no RS, tenho bacharelado pela UFRGS e licenciatura em matemática. Nas duas escolas que leciono consegui mudar os planos de estudo e acrescentar no terceiro trimestre para os terceiros anos, Física Moderna. li que você é favor dessa ideia.

Sim. Quando eu era professor na EPCAR, da Aeronáutica, em Barbacena, o segundo semestre da terceira série (o 6º do curso) era todo dedicado à Física Moderna. Mas isso foi nas décadas de 1960 e 1970. E lá eles não se importavam com o que caia ou não nos vestibulares. O livro adotado era o "Advanced Topics Suplement", do PSSC, que nós mesmos traduzimos e publicamos internamente. Os outros quatro volumes do PSSC eram dados nos cinco primeiros semestres, dessa forma: o volume 1 no primeiro semestre, o volume 2 no segundo semestre, o volume 3 no terceiro semestre e metade do quarto e o volume 4 na outra metade do quarto e no quinto semestres. Funcionava que era uma beleza. Tínhamos todos os kits do laboratório com uma réplica para cada dois alunos, que faziam práticas semanais em laboratórios com mesas para dois alunos. Todos os mais de 50 filmes foram dublados por nós e eram passados em películas de 16 mm. Nos tópicos avançados se estudava conservação do momento angular, cinemática relativística, física quântica, física atômica, física nuclear, física do estado sólido e física estatística. Para mim só faltava astrofísica e cosmologia. Tentei implantar isso em outros colégios mas não consegui aceitação. Não só a física moderna mas o método da redescoberta, pelo qual as experiências eram feitas antes das aulas teóricas e, nelas, os próprios alunos descobriam as leis. Para isso as práticas têm que ser semanais e feitas pelos próprios alunos. No máximo com três por equipe, de preferência dois. Senão uns ficam à toa. Um trimestre já é alguma coisa para se ver Física Moderna. Fico contente em saber que há lugares em que isso é feito.

Podemos dizer que algo é difícil por ver esse acontecer menos em relação às outras possibilidades de um "assunto". Mas não impossível, pois nunca viveremos tempo infinito para testar a impossibilidade total. Dizer que algo é difícil, mas não impossível, é uma pressão para alguém tentar algo?‎

Difícil, no caso de uma ação a ser realizada, significa que ela requer muito trabalho, muito tempo, que envolve procedimentos complicados e que requer muito conhecimento e muita habilidade para ser executada. Isso vale para a ação de se aprender alguma coisa. Se se trata de algo que se tem a intenção de aprender, deve-se dedicar a isso, mesmo sendo difícil. Impossível é quando há alguma barreira intransponível que, não importa o esforço que se faça e a dedicação que se tenha, não se logra conseguir realizar. Por exemplo, uma pessoa cega não consegue ser endodontista. Em geral, o aprendizado de algo, por mais difícil que seja, não é impossível.

O que você acha de pessoas pouco inteligentes? Você as odeia? Eu estava lendo um livro e não estava entendendo, aí peguei outro e não tinha resposta dos exercícios. Consegui responder 1 questão das 5 primeiras. Era um livro de Estatística Básica. A culpa sempre é do aluno?‎

Claro que não odeio ninguém por não ter muita inteligência ou por ser feio, gordo, magrelo, ignorante, deselegante, maleducado, pobre ou o que seja. Mas repudio quem seja cruel, malvado, injusto, desonesto, espertalhão, corrupto. Mesmo que seja inteligente, culto, elegante, rico ou bem-educado. Quem não consegue acompanhar e entender um livro desses, em geral, tem uma deficiência em sua formação escolar que vem desde o Ensino Fundamental. Não só em matemática, mas em português, para entender o que se lê. A culpa é tanto da própria pessoa quanto dos pais (ou outros cuidadores) e dos professores. Eu a reputo mais aos professores. Eles é que têm a obrigação de desenvolver nos alunos o interesse pelo estudo e convencer os pais a incentivar e ajudar os filhos em seu progresso escolar. Uma vez que o estrago já foi feito e se tenha consciência disso, há que se remediar. Mas não é fácil essa "recherche du temps perdu". A dedicação ao estudo tem que ser muito maior. Há que se pegar mais de um livro e se debruçar sobre eles frase por frase, buscando o entendimento de cada uma antes de prosseguir. Mas não é impossível. Só difícil, trabalhoso, cansativo e demorado.

E individualmente?

Do mesmo modo. Sou uma pessoa de índole e convicções completamente teóricas. Não sou nada prático. E considero que é assim que deva se ser. Há quem discorde de mim, mas é assim que penso. Toda ação deve ser conduzida do modo que a teoria prescreva. Mesmo que seja mais difícil, mais trabalhosa, mais demorada e menos eficiente. Mas será sempre mais eficaz e perfeita. E, para mim, eficiência não tem valor. Eficácia é que tem. A eficiência deve ser buscada, desde que não comprometa a eficácia e a perfeição.

Professor queria saber sua opinião sobre a religião satânica, queria entender porque algumas pessoas escolhem segui-la. Porque são más ou porque não se sentiram ''satisfeitas'' com o cristianismo ?‎

Não que sejam más, necessariamente. O que elas querem é alguma forma de conseguirem resultados vantajosos para si, por meio da pretensa interveniência desses supostos espíritos, que teriam a capacidade de agir sobre o desenrolar dos acontecimentos. Claro que isso não procede, mas há quem creia. Muitas vezes o que se deseja, mesmo que não seja uma maldade (mas pode ser), não seria algo a que Deus atenderia por meio de preces. Daí recorrerem a Satanás. Isso não se refere ao cristianismo, mas a qualquer outra religião, como a islâmica, a hinduísta, a judaica ou outra qualquer. Todas elas consideram que Deus (ou os deuses) atendam a pedidos razoáveis e bem intencionados. Pedidos gananciosos, invejosos, vingativos ou outros que tais seriam atendidos por espíritos malignos. Isso tudo, no entanto, é uma completa enganação. Nenhum pedido a nenhum agente sobrenatural é atendido, nem ao demônio nem a deuses. Simplesmente porque essas entidades não existem.

Professor, o senhor acha que todos nós nascemos com a mesma capacidade de aprender coisas e executar tarefas? Ou cada pessoa nasce predisposta à ser boa em algo?

Não nascemos com a mesma capacidade para tudo. Uns nascem com mais capacidade para algumas coisas do que para outras. Alguns nascem com grande capacidade para muitas coisas e outros com baixa capacidade para quase tudo. Isso é uma questão genética que não depende só dos ancestrais imediatos, mas de toda a ancestralidade, bem como outros fatores, como erros de transcrição do DNA que podem resultar em condições favoráveis ou desfavoráveis. E, mesmo, condições traumáticas no período de gestação, como o uso de drogas pela mãe. Todavia, sempre é possível aprimorar a capacidade mental pelo exercício neuróbico, que propicia a formação de dendritos e ligações sinápticas e, principalmente, se levados adiante na tenra infância, a evitar a dissolução de muitas ligações, que são extremamente mais abundantes nessa fase.

Você já deixou de agir por falta de apoio?‎

Muitas vezes. Por exemplo, uma de minhas propostas, que venho insistindo há mais de quarenta anos, é o ensino de Física Moderna (quântica, atômica, nuclear, relatividade) no Nível Médio, mas nunca consegui apoio das direções dos colégios, pois é um assunto que não cai nos vestibulares. No entanto é importantíssimo para que as pessoas tenham uma compreensão do mundo em que estão inseridas. Outra coisa que também proponho é o ensino de derivadas e integrais no Nível Médio, também sem apoio nenhum. Como a fronteira do conhecimento a cada dia fica mais distante, é preciso que alguns tópicos abordados no nível superior desçam para o nível médio, que alguns do nível médio desçam para o fundamental, que alguns do mestrado desçam para a graduação e que alguns do doutorado desçam para o mestrado. Com as modernas técnicas pedagógicas é possível que temas mais difíceis e complicados possam ser vistos em níveis mais elementares de aprendizado. Para isso é que existe a didática e a pedagogia.

O que você acha do curso de Química?

Pessoalmente não sinto grande fascinação por química, mesmo que, no científico, era a ciência em que eu sempre tirava as melhores notas. Porque achava muito fácil. Gosto mais de Física e Matemática. Mas não é que eu desdenhe a Química. Possivelmente eu até tenha mais interesse por Química do que a maioria das pessoas. É que não é o assunto de meu maior interesse. Mas acho que é uma ciência que pode propiciar satisfação a quem a ela se dedique, por gosto. E que pode, sem dúvida, dar uma grande contribuição para a melhoria da qualidade de vida da humanidade.

Técnico e Tecnólogo são a mesma coisa?

Não. Técnico é uma profissão de nível médio. Tecnólogo é de nível superior, mas de cursos de curta duração.

É possível que a energia escura ou matéria escura que compõem a maior parte do universo seja inteligente?

Não vejo como. A inteligência decorre da complexidade de um sistema, adrede estabelecida. A matéria escura é composta de partículas exóticas, que não se estruturam em átomos e a energia escura é só um campo não estruturado. Nenhum desses sistemas exibe complexidade suficiente para gerar inteligência.

Professor, estou no 3o ano do ensino médio, e quando acabar, pretendo fazer uma faculdade particular, e pretendo especializar-me em astro-física, porem a astro-física não é muito valorizada aqui no Brasil, pelo oque eu sei. O que devo fazer?‎

Astrofísica não é um curso de graduação. É uma pós-graduação (mestrado e doutorado). Para fazê-la é preciso fazer a graduação em Física ou Astronomia. Mas eu recomendo Física. Você pode fazer no INPE, na UFRGS, no IAG (USP), no Observatório Nacional. São os que conheço. O ideal é fazer doutorado no exterior. É bom fazer um curso de graduação (bacharelado) em Física numa instituição bem conceituada, como USP, UNICAMP, UFRJ, UFMG, UFRGS, UnB ou outras que tais. Pode fazer na UFV também que é bom. Astrofísica é bem valorizada. Só que tem poucos empregos. Você vai ter que trabalhar em universidades e institutos de pesquisa. Mas se você for mesmo muito bom de serviço, certamente conseguirá trabalho.

As "ondas gravitacionais", caso existam, possuem poder de ionização?

Não. Porque elas atuam igualmente sobre todos os corpos. Isso faz parte do "Princípio da Equivalência", que pode ser verificado pelo fato de que todo corpo, independentemente da massa, sofre a mesma aceleração por parte da gravidade. Para haver ionização o campo precisa separar as cargas positivas das negativas, isto é, empurrá-las em sentidos opostos. E a gravidade puxa tudo no mesmo sentido. O que a "onda gravitacional" faria é provocar uma vibração em bloco do corpo todo. Isso é testado em grandes cilindros perfeitamente circulares com sensores que verificam se ocorre alteração periódica na circularidade, com achatamentos e alongamentos.

Existe alguma diferença cognitiva, psíquica ou criativa entre escrever de próprio punho e escrever através de um teclado QWERTY?

Sim. O comando nervoso para fazer o desenho de cada letra é muito mais elaborado do que o comando de premir a letra no teclado. Um aprendizado, para ser mais eficaz, deve envolver, conjugadamente, várias áreas cerebrais: Audição, visão, locução e redação. Se possível, também, a realização prática de alguma atividade manual relacionada. E a redação é mais complexa, neurologicamente falando, quando é manuscrita. Do mesmo modo que a locução precisa ser em voz alta, para envolver os comandos dos músculos da fala.

Mas qual vai ser o sentido disso tudo? A terra vai acabar e ai? Não vai mais haver ninguém pra falar algo, pesquisar, etc. qual a lógica disso?

Não tem lógica e não tem sentido nenhum. É simplesmente o que vai acontecer em decorrência da evolução natural dos acontecimentos. A existência da vida na Terra é uma efemeridade, em termos de tempo cosmológico, surgiu, perdurou por uns sete bilhões de anos (estamos na metade) e sumiu. Do mesmo modo que deve ter acontecido, pode vir a acontecer e deve estar acontecendo em outros planetas do Universo. Enquanto isso, o Universo nem se importa com existir ou não vida em algum lugar. Assim também é a vida de cada um de nós. Não tem razão e nem propósito nenhum. Existimos porque calhou de certo espermatozoide fecundar certo óvulo que fez surgir a nossa pessoa. Poderiamos não ter surgido. Mas surgimos. E somos únicos no Universo, cada um de nós. Esse fato é tão singular que não se pode desperdiçar tão imenso privilégio (o de existir e ter consciência) levando uma vida fútil, sem significado e não gratificante. É preciso que coloquemos um sentido, dado por nós mesmos, à nossa vida, para que essa preciosidade não se desperdice. Porque ela não se repetira e, ao morrermos, acabaremos completamente.

Já presenciei a mesma situação que o usuário ''Valério Straütenbach Soares'' relatou, meu professor de arte disse que a evolução é a maior bobagem, pois a gente não ''vê um macaco evoluindo'', que se fosse assim todo mês teriam que abrir o zoológico pra novos humanos saírem(continua...)‎e que Deus criou as pessoas como elas são, vários alunos concordaram e começaram a tirar sarro de quem acredita na evolução, achei um absurdo vindo de um professor, ''ensinar'' de forma errada e influenciar os alunos a acreditar em algo divino, envés de abrir-lhes a mente pra questionar sobre.

Essa concepção revela uma total ignorância sobre o fenômeno evolutivo. Para começar, quando uma espécie evolui a partir de outra, isso acontece com parte de seus membros, mas parte continua a existir como a espécie original. Depois a caracterização de uma nova espécie só se dá ao longo de dezenas ou centenas de milhares de gerações, o que envolve milhões de anos. Cada filho é da mesma espécie do pai e da mãe, mas o descendente dezenas de milhares de gerações depois já pode não ser, uma vez que as mudanças são graduais. Raramente pode ocorrer uma mudança brusca, que faça o filho não ser da mesma espécie do pai. Quando alguém disser tais tipos de bobagens, peça-lhe que, primeiro, estude o assunto.

vc sabe como o mundo ira acaba ?‎

Depende do que você chama de "mundo". A espécie humana se extinguirá dentro de alguns milhões de anos, quando substituída por outras que evoluirão até que não sobre mais nenhum humano. A vida toda no planeta Terra se acabará quando o Sol se tornar uma gigante vermelha, dentro de poucos bilhões de anos e ficar tão quente que evaporará toda a água da Terra, incluindo os oceanos. O próprio planeta será pulverizado e gaseificado quando o Sol explodir, dentro de uns cinco bilhões de anos. O Sol se transformará em uma anã branca que depois se apagará e virará uma anã negra. A matéria do Universo se acabará quando a expansão cósmica chegar a um nível tal que romperá as estruturas subatômicas e transformará todas as partículas elementares em campo puro. O Universo, contudo, prosseguirá existindo indefinidamente sem matéria e nem radiação, preenchido só por campo, cada vez mais rarefeito.Depende do que você chama de "mundo". A espécie humana se extinguirá dentro de alguns milhões de anos, quando substituída por outras que evoluirão até que não sobre mais nenhum humano. A vida toda no planeta Terra se acabará quando o Sol se tornar uma gigante vermelha, dentro de poucos bilhões de anos e ficar tão quente que evaporará toda a água da Terra, incluindo os oceanos. O próprio planeta será pulverizado e gaseificado quando o Sol explodir, dentro de uns cinco bilhões de anos. O Sol se transformará em uma anã branca que depois se apagará e virará uma anã negra. A matéria do Universo se acabará quando a expansão cósmica chegar a um nível tal que romperá as estruturas subatômicas e transformará todas as partículas elementares em campo puro. O Universo, contudo, prosseguirá existindo indefinidamente sem matéria e nem radiação, preenchido só por campo, cada vez mais rarefeito.

Meu professor de filosofia falou hoje na aula que a teoria da evolução está indo água abaixo, pq segundo ele "como os macacos não evoluiram se a gente evoluiu" e ele disse também que o Big Bang é uma bobagem assim como a criação divina, pois do nada tudo "explodiu". Eu resolvi não contestar ele na a‎

Seu professor é uma pessoa ignorante. Não pode ser professor de Filosofia. Tem que se inteirar de ciência. Evolução e Big Bang não são opiniões e nem hipóteses. São teorias bem estabelecidas. Claro que podem vir a ser modificadas. Claro que os macacos evoluíram. Todos os macacos, bem como todos os seres vivos da atualidade evoluíram a partir de outros que os precederam. Isso nos inclui. Quanto ao Big Bang, ele não é o surgimento do Universo e sim o início de sua expansão. E esse surgimento não se deu a partir "do nada", pois não existia "o nada", para se transformar no que existe. O que existe surgiu "de nada", pois não havia coisa alguma para se transformar no que existe. Se assim não o for, então tudo o que existe sempre existiu. Mesmo as pessoas que acham que esse surgimento foi um ato de criação feito por alguma entidade onipotente extrínseca ao Universo consideram que essa entidade promoveu o surgimento de tudo sem que fosse proveniente de nada. Fale com ele para entrar aqui no Ask e discutir comigo.

Você é a favor de leis do uso de cinto de seguranças e do bafômetro?

Sim. Dirigir veículos automotores é uma atividade de grave responsabilidade que não pode ser feita sem que a pessoa esteja em pleno domínio de suas capacidades neurológicas, além de ser completamente habilitado para fazê-lo. Por outro lado, sabe-se que a falta do uso do cinto de segurança é um elevado fator de danos físicos e de morte em acidentes. Portanto acho perfeitamente legítimo que seu uso seja obrigatório, bem como a aplicação do etilômetro pela polícia, inclusive, podendo ser considerado alcoolizado quem se recuse a se submeter ao teste.

O que a mulher é para o homen ? O Homen nescessita mesmo de uma mulher para ter sucesso ?‎

A mulher e o homem são os dois sexos e os dois gêneros da espécie humana que necessitam um do outro para que a espécie sobreviva. Isso é biológico. Mas, por outro lado, socialmente falando, um necessita do outro para que a vida em sociedade possa ser conduzida satisfatoriamente. Mas não se pode dizer que homens necessitem de mulheres para se realizarem profissionalmente e nem vice versa. A maior necessidade que um tem do outro é afetiva e a realização afetiva tem grande impacto sobre os demais aspectos da vida.

Ernesto, qual sua opinião sobre o porte de armas? acha que a justificativa de proteção, em legitima defesa, são argumentos fortes para se legalizar?

Mas o porte de armas é legalizado. Quem tiver uma razão justificada pode ter. A questão é se se poderia ter armas sem justificativa. Isso eu acho que não. Mesmo que se diga que a população ficará à mercê dos bandidos, que terão as armas sem justificativa mesmo, acho que um incremento generalizado do porte de armas seria mais perigoso. Mas esta minha opinião ainda não é completamente bem estabelecida, pois ainda não me debrucei sobre o assunto para pesquisar e refletir.

Algum dia a Ciência poderá voltar a ter aquele sentido de exatidão que possuía antes do século XX? Como por exemplo, o sentido de máquina que a física clássica atribuía ao Universo? E pq?

Não. Aquela concepção era errônea. A descoberta da incausalidade e do indeterminismo intrínsecos dos fenômenos fundamentais da natureza é definitiva. Não se vislumbra nenhuma possibilidade de que isso venha a ser revogado. O comportamento quântico é inerente à natureza em seu nível mais elementar e todos os outros níveis, em última análise, reduzem-se a ele.

Quando esta chovend vc se molha mais parado ou correndo? Ou a mesma coisa?‎

Se a chuva estiver caindo na vertical, correndo molha mais, pois, então, em relação a você, ela virá oblíqua. Mas se ele estiver caindo oblíqua e você correr no sentido oposto ao que ela cai, ela cairá verticalmente sobre você, molhando-o menos.

Quando uma estrela perde todo seu combustível, e começa a cair sobre si mesma, o que acarretaria uma super nova e talvez um buraco negro. Essa Super nova rasga o tecido do espaço-tempo e por isso se forma um buraco negro?‎

Não. O buraco negro não rasga o tecido do espaço-tempo. Ele está normalmente imerso nele. E nem toda supernova resulta num buraco negro. Depende do valor da massa do caroço remanescente. Se ele for menor do que 1,4 massas solares, formará uma anã branca. Se estiver entre 1,4 e 2,7 massas solares, formará uma estrela de nêutrons. Se estiver acima de 2,7 massas solares, formará um buraco negro.

http://ask.fm/wolfedler/answer/109412024861 Uma bolha de ar na água ou a bolha de sabão são redondas pelo mesmo motivo?‎

Não. Nesse caso a esfericidade advém não da maior compacticidade (existe essa palavra?) da esfera para um dado volume, mas da menor superfície da esfera para um dado volume. Pois a forma é resultante da tensão superficial da película líquida da bolha ou da superfície interna da água em torno da bolha de gás nela imersa. A "compacticidade" significa que se se somar todas as distâncias entre todos os pares de pontos de certo volume, essa soma será menor se esse volume for esférico. Outro fato é que para todas as formas sólidas com dado volume, a que tem a menor superfície externa é a esfera. No caso da gravidade o formato esférico provém de que ela atrai (busca aproximar) um ao outro, todos os pares de elementos de massa do volume de um corpo. No caso das bolhas, a força é entre os pares de elementos da superfície e não do volume. O resultado é o mesmo, mas a razão é outra.

O pensamento dos seres-humanos segue uma lógica ou é livre? Como essa lógica surgiu?

O pensamento espontâneo não é lógico. A lógica é uma disciplina adquirida por treinamento. A lógica surgiu da observação e da reflexão sobre como se tirar conclusões a partir de pressupostos. Até hoje, a lógica ainda é um tema em desenvolvimento, que, a cada dia, incorpora novas considerações sobre a forma de raciocinar corretamente.

É verdade que as maiores Guerras e Genocídios foram praticados pela Ideologia Ateísta?

Não. Os maiores genocidas foram Mao Tsé Tung, Stálin e Hitler. Todos mataram em nome do poder que almejavam. Mesmo que o dito "Comunismo" de Mao e Stálin também fosse ateísta, não foi por causa do ateísmo que ele matou tanta gente. Foi principalmente por razões políticas e econômicas. Quanto a Hitler, ele era católico e, parte de sua justificativa para o genocídio judeu foi por eles serem o povo que matou Jesus. Mas as religiões, no total, também mataram gente à beça, seja em nome de Jesus, de Maomé, de Yahweh ou de outro que fosse.

Você acha que o legado que a Copa deixará aqui no Brasil compensará todas os gastos e investimentos feitos?

Não. Para mim vai dar um grande prejuízo. Especialmente considerando o dinheiro que está sendo desviado, não só para o bolso dos que se locupletam dessas obras todas mas, principalmente, porque esse dinheiro não está sendo aplicado onde precisava ser. O lucro do turismo não vai compensar. E, depois, esses estádios todos e as demais obras se tornarão elefantes brancos, completamente inutilizados.

http://bit.ly/1g4k8v9 Acha que esse tipo de atitude é válida e louvável, se impedir que o mundo se torne uma grande Sodoma, ou deve-se permitir que a humanidade tenha a liberdade de descer o mais baixo possível em sua condição?‎

É preciso distinguir as coisas. Concordo com ele quanto ao fato de que perseguições religiosas não podem ser admitidas contra nenhuma religião. Nem contra cristãos em países islâmicos nem contra muçulmanos em países cristãos. Nem contra judeus, nem contra espíritas, umbandistas etc. Mas não se pode admitir religiões que promovam ações que firam direitos e a ética, como sacrifícios de animais, por exemplo (sem falar, até, de humanos). Sobre a depravação de costumes, também concordo que seja preciso coibi-la. Mas não concordo de que homossexualidade seja depravação, de modo nenhum. Homossexualidade é uma forma de amor. Nem mesmo o poliamor e a poligamia são depravações. A laicidade da sociedade é outro valor que reputo de alta significância. É ela, inclusive, que garante a liberdade de expressão para todas as religiões. Incluindo para a falta delas. Não ter religião, absolutamente, não se configura em amoralidade., Ética e moral são valores humanos e não religiosos.

o senhor não acha que dar consciência a uma maquina pode ser perigoso como aconteceu em varios filmes de ficção cientifica como "o exterminador do futuro'? ou em outros que maquinas foram criadas para criar a paz e decidiram aniquilar a raça humana?‎

Pode. Mas isso fatalmente acontecerá, algum dia. Como a Bomba Atômica e, aliás, todas as descobertas e invenções que o engenho humano conseguiu fazer, desde uma faca até naves espaciais, passando pela pólvora, o avião, e quase tudo que tanto pode ser usado para o bem quanto para o mal. Assim, não acho que se deva evitar nem o conhecimento nem a tecnologia pelo medo do mal que possa fazer. Pode sim e fará. Mas o bem que faz é muito maior.

Máquinas podem vir a ser conscientes?‎

Acho que sim. Não há empecilho nenhum, exceto a falta de competência tecnológica, que, um dia, será superada

concorda com a frase de "Viver é diferente de estar vivo"?

Isso depende de como se conceitua cada expressão. Pode ser que seja ou que não, conforme cada um interprete o significado. Veja minha resposta anterior.

o que o senhor acha sobre a teoria de que NAO existe os buracos negros?‎

Não é isso que o Hawking disse. A imprensa distorce as informações. O que ele disse é que há a possibilidade de que algo consiga ser emitido a partir de um Buraco Negro. Mas a entidade existe sim. Só que suas propriedades estão com uma proposta de revisão, ainda não verificada.

Qual a diferença de viver para sobreviver?‎

Sobreviver é, apenas, não morrer. Viver é, além disso, passar a existência de forma gratificante e significativa.

Como a gravidade torna os planetas esféricos?

Toda força atrativa central age de modo a fazer com que as partes que se atraiam se disponham o mais próximo possível umas das outras. Se elas podem se mover, como as moléculas de um gás, elas se deslocarão até que isso ocorra. A forma esférica é a que propicia, para uma dada massa, a forma mais compacta de acomodação do conteúdo (isso pode ser demonstrado como um exercício de cálculo variacional). Então, qualquer massa de gás, em razão da gravidade mútua de suas partes, se disporá de uma forma esférica. Todavia, as partes sendo sólidas, se a massa total não for bem grande, elas podem se aderir de uma forma não esférica, como as luas de Marte e a maioria dos asteroides, exceto Ceres. No caso de haver um momento angular total da massa não nulo, a esfera será formada em rotação, o que acarretará uma deformação pelo efeito centrífugo, levando o astro a assumir a forma de um esferoide oblato.

Vc disse que nao gosta de LETRA GRANDE, mas pq escreveu em letra grande na sua Bio?‎

As letras maiúsculas podem ser usadas para revelar algum destaque. Mas não na totalidade do texto, pois, nesse caso, não mostram destaque para nada. O mesmo se aplica ao negrito, ao itálico e ao sublinhado. Como aqui não se tem negrito, sublinhado nem itálico, só restam as maiúsculas para mostrar algum destaque.

professor eu tenho uma duvida, um buraco negro é um evento natural? se for qual a finalidade dele no universo?‎

Sim. É natural. Mas nada na natureza tem alguma finalidade. Tudo o que existe, existe porque surgiu, por acaso. Não há propósito para os buracos negros. Quando algo parece que tem propósito é porque aconteceu de servir para algo. Mas o surgimento nunca tem a pretensão de ter alguma utilidade. Somente artefatos planejados e produzidos por entidades inteligentes é que possuem propósitos. E a natureza não é inteligente.

Tem uma gama de dados, ideias e informações na Deep Web que não são divulgadas na Web comum. Não relacionadas a coisas hediondas e repugnantes.‎

Pode ser. Mas, para mim é um tipo de esoterismo. Ou seja, algum conhecimento que se considera deva ser restrito a certa categoria de pessoas. Sou totalmente contra isso. Acho que todo conhecimento tem que ser divulgado para o povo todo saber. Então, porque mantê-lo em sites inacessíveis a todo mundo? Nem sei como se acessa essa deep web. Qual a razão de ser "deep"? Para mim algo que se quer escondido é porque é ilegal.

Por que tem umas pessoas que gostam de criticar seu ask na sua opinião? Na minha foi um dos únicos que tem algum proveito intelectual.

Não tenho conhecimento das críticas negativas a meu Ask. Quem as faz, não as passa para mim. Gostaria de saber, para me defender. Certamente que há quem não concorde com minhas ideias ateístas e anarquistas. Quanto ao que eu digo sobre cosmologia, evolução, física e outras ciências, não são opiniões, mas conhecimentos estabelecidos até o momento. Agradeço a gentileza de sua apreciação.

Se o Universo é infinito e a matéria sempre existiu e sempre existirá[ em formas diferentes é claro], então a humanidade e suas ocupações são mesquinhas face a isso?‎

Que o Universo seja infinito é o que as informações mais atuais mostram, mas que sempre existiu não. O que se sabe, por enquanto, é que ele teve um surgimento, antes do qual não havia nada. Mesmo assim, a humanidade e suas preocupações, de um prisma cósmico, são completamente irrelevantes. Mas, como somos humanos, para nós é relevante. E, por mais mundos que haja no Universo, só temos acesso, por enquanto, a este. Temos, pois, que cuidar dele muito bem e aprendermos a viver em harmonia dentro dele. Ou seja, acabar com essas disputas idiotas por causa de religião e de recursos econômicos, compartilharmos tudo uns com os outros, acabarmos com a doença, os crimes, a ignorância, a cobiça, a competição e todos os males e nos devotarmos a construir uma mundo fraterno, justo, próspero, aprazível, livre, igual e feliz para todos. Isso é um esforço que temos a obrigação de nos dedicar, com toda a generosidade. Bem como a preservar o ambiente para garantir a sobrevivência de todos neste rincão. Todos, não só os humanos.

Eu sei que você sempre gostou de aprender, mas com que idade começou a se aprofundar em assuntos intelectuais?‎

Desde que me entendo por gente. Aprendi a ler com 4 anos e, já com 7 anos, lia romances e livros de divulgação científica. Com uns 11 ou 12 anos comecei a estudar filosofia. No ginásio eu estudava história e geografia nos livros de faculdade do meu pai, que era professor disso.

Oq se ve na deep web e a prova que anarquia nunca vai acontecer, pessoas como aquelas que comem gente, fazem sexo com recém nascidos, matam por prazer, torturam e estupram crianças com a maior crueldade que se possa existir.Essas pessoas não tem concerto.Sempre vão existir pessoas más e nojentas‎

Você se baseia na humanidade atual. Pense para daqui a milênios. Depois da erradicação de toda a pobreza, de toda a doença, de toda a ignorância, por um processo educativo de muitas dezenas de duração, se poderá acabar com tudo isso e criar as condições propícias para não se ter governo, estado, lei e nada disso e todo mundo ser justo, honesto, bondoso, ordeiro, diligente, desprendido, em suma, virtuoso. A anarquia não é um meio para se conseguir nada. É a culminância do processo civilizatório da humanidade. Considerando que a humanidade só tem 200 mil anos e ainda pode durar alguns milhões de anos e, considerando ainda, que a civilização tem crescido e a tendência para a liberdade com responsabilidade é cada vez maior ao longo dos últimos milênios, é de se esperar que tal estágio supercivilizado possa ser alcançado sem que se espere muitos milênios.

E onde fica localizado este buraco negro que você disse que existe no universo? Ah, e você pode nos dar mais informações sobre a esfera que há no centro da via lactea?‎

Esse buraco negro que mencionei está no centro da Via Láctea, a galáxia a que nosso Sol pertence. Grande parte das galáxias possui um buraco negro em seu centro. A esfera que eu mencionei é uma esfera imaginária que contenha toda a matéria desde o centro da galáxia até alguma distância dele. Todas as estrelas, quer dos braços espirais, quer dos aglomerados globulares das galáxias, orbitam o total de matéria contido nessa esfera. A esfera varia com a distância e é puramente imaginária.

Toda estrela tem planeta a orbitá-la? Qual é o conceito de Planeta? Pq Plutão deixou de sê-lo?

Não. Só a maioria. Planeta é um corpo celeste de dimensões insuficientes para gerar energia por si mesmo, em alguma época de sua evolução, mas suficiente para ter se tornado esférico pela própria gravidade e capturado toda matéria planetesimal da região circundante. Além do mais só é chamado de planeta um corpo assim que orbite uma estrela. É o caso da Terra e de sua Lua, que constituem um sistema binário de planetas. Astros com dimensões planetárias (que vai até 13 massas de Júpiter) mas que não orbitem estrelas são chamados de estrelas sub-anãs marrons. Plutão passou a ser considerado um "planeta anão", porque, apesar de ser esférico, sua massa não é suficiente para promover a limpeza de planetesimais de suas região, já que se encontra no Cinturão de Kuiper. Como Ceres, que está no Cinturão de Asteróides, entre Marte e Júpiter. Veja isto:
http://pt.wikipedia.org/wiki/Planeta
http://pt.wikipedia.org/wiki/Planeta_an%C3%A3o

Qual a relação da Matemática com o mundo físico? Este cessaria de existir se não houvesse aquela para o descrever, sustentar?

A Matemática fornece ferramentas que permitem construir modelos descritivos do comportamento do mundo. Mas o mundo não depende da Matemática para ser como é. Ele é como é por conta própria, haja ou não Matemática para descrevê-lo, bem como qualquer ciência.

Existe um buraco negro no centro da via lactea que coloca toda a via lactea em sua orbita?

Existe um buraco negro, mas ele tem a massa de quatro milhões de sóis, enquanto a Via Láctea tem a massa de um trilhão de sóis. Logo ele é pequeno comparado com ela. Cada estrela da Via Láctea orbita toda a matéria que existe em uma esfera de raio igual à sua distância ao centro da galáxia.

Quanto tempo leva para aprender inglês estudando 20 horas por dia?‎

Estudar 20 horas por dia a mesma coisa não vai acelerar tanto o aprendizado porque o cérebro vai se saturar. Não acho proveitoso passar de umas 6 horas por dia estudando o mesmo assunto e, mesmo assim, não seguidas e sim com dois períodos separados de 3 horas ou três de duas. Nesse caso acho que dá para aprender em uns três meses.

http://ask.fm/wolfedler/answer/109355574813 Entrevistei pessoas mais velhas que viveram na época. Nenhuma me disse que uma ditadura de fato ocorreu, apenas um Regime Militar e que esse não mudou a vida deles pra pior. Pelo contrário, 100% dos meus entrevistados relatam que a vida era muito melhor.

A vida era melhor porque a população era menor. Quem não tinha envolvimento com nada na política passou o período tranquilamente. Mas quem era da oposição foi perseguido, torturado e morto. Isso é ditadura sim. Os presidentes não eram eleitos pelo povo. O congresso foi fechado muito tempo. Eu não estou dizendo que a situação atual esteja boa e nem que o governo atual está bom. O que estou dizendo é que o regime democrático é que é o melhor. É uma questão de princípio.

Eu fico muito em dúvida sobre tal assunto, pois me parece que físicos não têm muitas áreas de atuação como outras profissões.‎

Físicos atuam em Universidades e Institutos de Pequisas. Se forem Físicos Aplicados, também podem atuar em indústrias, especialmente eletrônica e de materiais. O trabalho nas Universidades e Institutos requer aprovação em concurso. Então é preciso ter um currículo destacado, com um excelente histórico escolar, muitos trabalhos publicados durante a pós-graduação (mestrado e doutorado), especialmente em revistas de renome internacional e boas recomendações dos professores e orientadores. Além de, normalmente, ser o mais bem classificado na prova de conhecimentos e didática, no caso de Universidades.

Qual sua opinião a respeito do crescente número de pessoas que pedem a volta do Regime Militar?

Que cometem a mais rematada besteira. Geralmente não viveram aquela época ou, se viveram, estavam do lado opressor, que se locupletava da situação. Neste caso, são pessoas abjetas. A liberdade e a democracia não podem ser suspensas para solucionar nenhum problema de segurança, de economia, de moralidade pública, ou o que for. Tudo tem que ser resolvido dentro da lei, sem nenhuma situação de exceção.

Ernesto, eu estou em dúvida sobre cursar Física ou Engenharia Mecânica.Caso eu opte por Física, terei que dar aulas para futuramente ser um pesquisador em determinada área?

Não necessariamente. Você pode fazer o bacharelado e entrar direto no mestrado ou, mesmo, no doutorado. Mas eu recomendo que faça a licenciatura e lecione. O magistério é importante para treinar a comunicação e um cientista também tem que ser um comunicador. Tem que ter didática. E, geralmente, todo cientista acaba lecionando no mestrado e no doutorado. E é muito ruim entender do conteúdo e ser um péssimo professor. Além do que o magistério é uma das mais nobres atividades a que alguém possa se dedicar na vida. Na juventude eu também pensei em ser Engenheiro Mecânico ou Arquiteto. Mas mudei para Física e Matemática porque cada vez mais fui me encantando com o conhecimento em si mesmo, independente de suas aplicações práticas. E a Física é a mais filosófica das Ciências. Acho, realmente, deslumbrante.

Estou curioso. Digamos que, por uma mínima chance, nós ateus estamos errados, e que após a morte descobrimos uma "vida espiritual" que somos incapazes de detectar nesse presente momento. A minha reação seria de gargalhadas, e a sua?‎

De espanto. Como poderia algo real não ser detectável, nem por indícios indiretos?

Professor, o curso de Física é muito difícil? O senhor acha que seria bom fazer Física e depois pós em Cosmologia ?

É muito difícil sim. Mas difícil não é impossível. Aliás, sua dificuldade é, inclusive, um dos seus atrativos. O que é fácil não tem graça. É muito gostoso estudar algo que seja bem complicado. Dá uma imensa satisfação, justamente, queimar a cuca. Cosmologia, Física de Partículas e Teorias de Campo, para mim, são as áreas mais interessantes da Física. Exatamente porque são as mais filosóficas e as mais profundas. Aquelas que cuidam do entendimento e da compreensão das estruturas e dos fenômenos mais fundamentais da natureza, responsáveis por tudo ser do modo que é. É fascinante. Como ser um compositor de sinfonias.

Professor, alguns assuntos que estudo com o tempo vou esquecendo e chegando até esquecer completamente. Isso é normal? Você tem algumas dicas de experiências para me oferecer?‎

Tudo o que você não considera relevante é esquecido mesmo. O único meio de não esquecer é ter um grande interesse pelo assunto, de modo que ele esteja sempre sendo revisitado. Se quiser se lembrar de tudo, tem que ter interesse por tudo. Mas não um interesse forçado e sim um interesse legítimo, que brota das profundezas de seu eu.

Acha possível viver-se uma vida e não possuir plena ciência do que és? Bom dia! Devido ao sumiço, como andas?

Bom dia! Mas não estou sumido. Continuo aqui, apenas com participação mais reduzida. Claro que é perfeitamente possível viver-se a vida toda sem saber, de fato, quem se é, não só individualmente, mas também como espécie. Aliás, a maioria vive assim. Ter a consciência clara de si e do que se é como um ser humano requer conhecimento e reflexão. Isso deveria ser propiciado pela escola, mas não o é. Infelizmente, mesmo agora que Filosofia é obrigatória, não se filosofa na escola. Apenas se aprende história da Filosofia.

Seria a ética e a moral subjetivas?Por quê?

Nem a ética nem a moral são subjetivas. Elas são sociais. Enquanto a moral é contextual em relação à época, ao local e ao extrato social, a ética é universal. A ética estabelece o que seja certo ou errado, bom ou mau, justo ou injusto. A moral estabelece o que é permitido e o que é proibido. Ambas concernem às ações. Mas nem sempre a moral acompanha a ética, como deveria. Todavia nem as prescrições morais nem as considerações éticas são individualizadas. Não existe algo que seja certo para mim e errado para você ou que me seja permitido e lhe seja proibido. Estas considerações e prescrições não dependem da pessoa a que se aplicam.

Pois um homem pode realizar o seu traba­lho com sabedoria, conhecimento e habilidade, mas terá que deixar tudo o que possui como herança para alguém que não se esforçou por aquilo. Isso também é um absurdo e uma grande injustiça. Eclesiastes 2:21 - Há fundamento.‎

Sim. Considero que herança seja o tipo da prática que deveria ser abolida. Não tem cabimento em uma sociedade justa. No caso de morte, se pagaria uma pensão para os filhos menores e se deixaria uma casa para morarem, se houver. O resto o governo recolheria. Mulheres e maridos não deveriam receber nada. Cada um que se sustente por si mesmo. Aliás, deveria ser impossível alguém acumular bens acima do necessário. No máximo uma casa e um ou carro. Só. Claro: mobília, roupas, louças, talheres, livros e esse tipo de coisa. Mas até um certo limite.

Professor você tem conhecimento que existem milionários, donos de aglomerados de igrejas e outras mídias anexadas as igrejas, esses caras por usarem a fé dos fiéis, para obter lucro é crime?

Claro que é crime. São estelionatários. Têm que ser acusados, processados, condenados e presos, bem como devolverem o dinheiro extorquido aos fiéis. Essas igrejas precisariam ser fiscalizadas pelo Ministério da Fazenda e pelo Ministério da Justiça. E seus adeptos advertidos pelo governo do fato de estarem sendo vítimas de extorsão.

Professor, a maioria dos alunos, assim como eu, têm dificuldade em interpretar as questões mesmo sabendo as formulas. Você teria algum conselho para anular essa dificuldade?

Isso é um problema de interpretação de texto que decorre da falta de mais leitura, bem como de redação. É um problema de português e não de matemática ou física. Isso não se resolve da noite para o dia. Requer muitas horas de leituras, anos a fio e muitas horas de redação. Mas pode melhorar treinando a leitura e interpretação de texto de problemas de física e matemática mesmo. Como, por exemplo, reescrever o enunciado do problema com suas próprias palavras e inventar problemas, dando a sua redação.

http://ask.fm/wolfedler/answer/109351288605 Entendi seu ponto de vista Professor, mas acho que não me expressei de maneira clara. Eu não perguntei se você concorda com o que o dono da frase lhe propós, mas sim se o que ele propõe é o que acontece no nosso cotidiano.

Isso depende do meio em que você vive. Comigo, por exemplo, não acontece. Talvez porque eu viva em um meio muito intelectualizado. Meus amigos, quase todos, são mestres e doutores, professores universitários e pesquisadores. Nesse meio, as pessoas aceitam o ateísmo normalmente. Inclusive tenho conhecidos que são pastores (mas não neo-pentecostais) e aceitam o fato de eu ser ateu sem me condenarem. Em outros meios pode ser que as pessoas execrem os ateus. Mas eu não sei, pois não tenho essa vivência.

Mas por qual motivo eu posso fazer somente a operação de soma de vetores e de produto por escalar para atestar se um subespaço vetorial é de fato um subespaço vetorial sem precisar atestar os demais axiomas?‎

Porque sendo um subespaço, automaticamente atenderá as propriedades do espaço. Só o que precisa ser testado é o fechamento, isto é, se qualquer resultado de operações com elementos do subespaço leva a um elemento também pertencente a ele.

Para verificar se um espaço vetorial é de fato um espaço vetorial, eu tenho que testar os oito axiomas ou por tabela, posso apenas fazer as operações de soma de vetores e do produto por escalar sem usar os demais axiomas?‎

É preciso testar, pois nem tudo que parece vetor e pode ser somado é um vetor. Por exemplo, as rotações podem ser representadas por algo do tipo de um vetor, com direção ao longo do eixo de rotação, com módulo igual ao ângulo de rotação e com sentido dado pela "regra da mão direita", ou do "parafuso de rosca direita". No entanto, tal objeto não é um vetor, pois não obedece à propriedade comutativa da soma. É chamado de um "pseudo vetor" ou um "vetor axial".

O fato que eu quero falar é que pelo jeito que vc falar se a pessoa não concorda com a homossexualidade ela é homofóbica e isso não é verdade.‎

Sim. Pode não ser. Todavia mesmo não sendo homofóbica, não vejo porque não aceitar a homossexualidade como algo perfeitamente normal. Não só no caso de ser uma orientação nata mas, inclusive, no caso de ser uma opção voluntária consciente, mesmo que divergente da orientação de nascença. Não só no caso da homossexualidade, mas quanto às demais orientações sexuais, como a bissexualidade, a assexualidade e a heterossexualidade. Há heterossexuais que optam por serem assexuais, como os sacerdotes católicos, e isso não causa estranheza. Do mesmo modo que há homossexuais que optam pela heterossexualidade e isso não é condenado.

Se algo pode vir a existir a partir de nada, então torna-se inexplicado o fato de que tudo o que observamos provém de algo. Mesmo as partículas virtuais no vácuo quântico ou os átomos excitados emitindo fótons possuem condições e provém de algo.De nada, nada se obtém, porque não existem propriedades‎

Uma vez que já existe universo, há leis naturais que esse universo observa, dentre as quais as leis de conservação. As leis naturais não são prescritivas e sim descritivas. Mas elas valem porque não há ocorrência em inobservância delas. Assim as transformações, mesmo as incausadas, se dão com o preenchimento de condições, dentre elas o fato de que o que surge, provém de algo pré-existente. Tal fato, todavia, não se aplica ao evento de surgimento do conteúdo total do Universo, uma vez que, não havendo nada, não há leis a serem observadas. Então não é preciso que algo surja tendo do que provir. Aliás, mesmo quem considere que o surgimento do Universo tenha sido um evento provocado por um agente extrínseco a ele (Deus), é forçado a admitir que esse agente provocou o surgimento de tudo o que existe sem que isso fosse proveniente de nada pré-existente. Pois, se houvesse algo de que Deus se valesse para criar o Universo, então esse algo já seria o Universo, que, na dita criação, estaria apenas sendo transformado pelo suposto Deus. A questão, pois, não é o fato do Universo ter surgido de nada. Isso é fatal. A questão é se surgiu espontaneamente ou provocadamente. E se foi provocadamente, o provocador não poderia fazer parte do Universo. A outra alternativa é que ele tenha sempre existido.

"Os cristões agem com tanta autoridade e imperialismo ao questionar-nos se somos ateus, como se um "sim" poderia ser o ápice para que não fossemos aceitos na sociedade atual por uma diferença dogmática, ou a falta dela. E por isso me recuso a afirmar que sou ateu." - Concorda com essa frase?‎

Discordo completamente. Porque eu teria que me preocupar com a opinião dos cristãos a respeito do meu ateísmo? Do mesmo modo que eu admito que eles possam ser cristãos, acho que eles têm que admitir que eu possa não ser. Como qualquer um de qualquer religião a respeito de qualquer outra ou da falta de alguma. Se não admitirem isso como um direito de cada um, então são pessoas intolerantes que não merecem a menor consideração de minha parte. Afirmo que sou ateu e acho que todo mundo que é tem que afirmar, do mesmo modo que todo mundo que professe qualquer religião tem que afirmar de qual seja. Como se afirma que se é torcedor de tal ou qual time de futebol ou de nenhum, que é o meu caso. É preciso afrontar a sociedade mesmo, para que as pessoas mudem o modo de pensar e admitam que os outros possam pensar de modo diferente delas, sem que isso se configure em nenhuma falta.

Algum dia, o senhor já considerou estudar design ou arquitetura, antes de se decidir pela física?

Quando eu era garoto eu pensava em ser arquiteto ou engenheiro mecânico. Só lá para os meus 17 anos é que mudei para a Física. Fui ficando cada vez mais teórico e menos prático. Passei a gostar do conhecimento por si mesmo, independentemente de suas aplicações.

Jesus disse para amar-nos uns aos outros como ele nos amou, morrendo por nós e nos salvando. O que você diria a um cristão que alega crer em sua religião por ela ser a única onde, ao invés do homem ir em busca de Deus, Deus busca o homem e aparece para ele, demonstrando seu inimaginável amor?‎

Sou fã desse ensinamento de Jesus. É assim mesmo que tem que ser. Temos que amar a todo mundo, seja quem for, seja o que for, tenha a religião que tiver. Por isso é que acho que as pessoas homofóbicas não podem se dizer cristãs, pois estão praticando o desamor pelos homossexuais. Mesmo que se considere que isso seja pecado (e não é), há que se amar aos pecadores também. Quanto ao fato de ter nos salvado com a sua morte, apesar de ser um ato de amor dele (a se considerar a veracidade dessa história toda), foi uma imensa crueldade de seu pai. Onde já se viu um pai amoroso exigir o sacrifício de seu filho para aplacar a sua birra com o pecado original? Porque não perdoou, simplesmente? E porque a humanidade toda teria que pagar o pecado só do Adão e da Eva? Essa história da redenção, para mim, é totalmente absurda. Esse fato, contudo, não credencia o cristianismo como religião verdadeira em relação às demais. O Islã também tem fortes argumentos para se considerar a verdadeira religião. Do mesmo modo que as outras. Não há nada, em nenhuma delas, que garanta que ela seja a correta e as outras erradas. Aliás, se existisse uma correta, ela deveria ser única, pois como é que Deus iria permitir que cada facção da humanidade achasse que sua religião fosse correta, estando equivocada. Como Deus da espécie humana toda, porque ele iria privilegiar o povo judeu com a verdadeira revelação e deixar os outros povos no erro. Esse não seria um Deus justo. A existência de várias religiões é uma dos maiores argumentos para se considerar que nenhuma seja certa, mesmo que haja Deus.

Olá, professor. Qual sua opinião sobre ateus que, muitas vezes inconscientemente, acabam assimilando a sua linguagem termos como "Ai, meu Deus", "Graças a Deus", "Credo", "Meu Deus do céu", etc? Você procura evitá-los?‎

Tais expressões já se tornaram idiomáticas, de modo que ao se pronunciá-las não se está, literalmente, expressando o que elas dizem. Não faço questão de evitá-las, mas, normalmente, não as uso. Como "Nossa Senhora!", "Virgem Maria!" e outras que tais.

Professor,como planetas começaram a girar ao redor de estrelasl?O que gera o movimento de rotação da terra

Eles já foram formados assim, a partir da matéria que foi expulsa da estrela num processo centrífugo. As estrelas, por sua vez, foram formadas girando porque, ao surgir um aglomerado fortuito de gás que começou a atrair mais gás das redondezas para si, esse gás já tinha alguma velocidade aleatória, de modo que não foi concentrado radialmente e sim apresentando alguma velocidade tangencial. Então a estrela surgiu com rotação. Ao se concentrar, apareceu o efeito centrífugo, que lançou parte de sua matéria para um disco em torno dela. Nesse disco, ocorreram outras concentrações, que formaram os planetas. Essas concentrações, por sua vez, também não eram radiais, de modo que os planetas já surgiram com rotação.

O que acha da teoria do design inteligente?‎

Completamente sem fundamento. Não há planejamento nenhum para a estrutura do Universo, da vida e da inteligência. Tudo surgiu por mero acaso.

Professor, o que pensas a respeito do tema e do lema da Campanha da Fraternidade deste ano? Se ainda não estiver inteirado, aqui está um link apresentando-a: http://www.portalkairos.net/cf2014/. Gostaria, depois, que você expusesse sua opinião no que tange ao escopo dessas campanhas como um todo.‎

Concordo plenamente com a validade do tema da atual Campanha da Fraternidade. Só não acho que seja um tema religioso e sim um tema referente aos direitos humanos de modo geral. Acho que as leis deveriam ser muito mais rígidas com esse crime, como escravidão e similares. Tipo, prisão perpétua com trabalhos forçados. E que a polícia precisaria atuar com muito mais eficácia para descobrir esse tipo de crime que tem que ser inafiançável e isento de qualquer benefício de redução de pena ou sursis (aliás, em minha opinião, qualquer outro - fiança é uma imoralidade). Acho que essas campanhas, em geral, são válidas, pois estimulam o espírito solidário dos católicos para com toda a população, seja católica ou não.

Existe alguma definição pra quem não tem nenhuma crença, não é ateu nem agnóstico? Tipo.. nem busca afirmação da não existência de divindades como no ateísmo e nem trata como incógnita do sobrenatural q não da pra afirmar nem negar como no agnosticismo? Exemplificando, se deus existe ou não foda-se.

Quem não tem nenhuma crença é incrédulo. Quem não se importa com o fato de existirem ou não deuses é o agnóstico. Os ateus são incrédulos. Não há como não ser nem ateu, nem agnóstico, nem crente. Ou se acha que deus existe, ou que não existe ou não sabe.

É possível aprender falar inglês em um ano?‎

Sim. Ou menos ainda. Depende da dedicação que ela tiver a esse aprendizado. Se se dedicar umas duas horas por dia, sem faltar nenhum, nem domingo nem feriado, acho que consegue em seis meses.

Você acha que existe um amor, para todos?‎

Não necessariamente. Isso é uma questão de coincidência. Uma pessoa pode ser objeto do amor de várias outras, como pode não ser de ninguém. Da mesma forma que pode sentir amor por várias outras ou por ninguém. Não há nada predeterminado a esse respeito.

Por que muitos big-bangs criando muitos universos é tão menos provável que apenas um big-bang criando este universo?

Porque não há indício nenhum da existência de outro universo. É uma conjectura gratuita. Da mesma forma que as pontes de Einstein-Rosen, os Buracos Brancos, os tachions e propostas similares. Elas advém de extensões teóricas das soluções matemáticas das equações que descrevem o comportamento do Universo e seu conteúdo. Mas isso não significa que correspondam a nenhuma realidade. Só podem ser aceitas se confirmadas observacional ou experimentalmente.

Esta evidente para vc que vc não vai sentir mais nada após a morte?

Não, mas os indícios são tão grandes que é inescapável considerar que sim. Não conheço nenhuma testemunho de quem já tenha morrido. Além do mais, como sentir algo sem que o cérebro esteja funcionando? Toda a neurociência mostra que a vida psíquica decorre do funcionamento do cérebro. Portanto, a morte é como um sono profundo sem retorno. Nem se sabe que se existe, no caso do sono profundo. E, na morte, não se existe mesmo.

Existe muita diferença entre técnico e graduado?

Sim. O técnico é treinado para executar e o graduado para criar e planejar. Essa é a principal diferença. Um engenheiro é um inventor. Um técnico é um executor de projetos e não um projetista

Por que confia nas pessoa? Não tem medo que elas te enganem ou te façam mal?‎

Porque é muito mais tranquilo viver assim. A maioria das pessoas é, de fato, de confiança. Algumas não. Mas é uma paranóia ficar desconfiando. Claro que isso pode causar algum prejuízo. Mas levar prejuízo não é tão ruim quanto viver sobressaltado. Causar prejuízo é que é terrível. Levar não.

Por quê a mídia chama Cuba, Vietnã e Coreia do Norte de "regime comunista", sendo que os mesmos não são nações comunistas?

Porque, historicamente, os socialistas, desde o fim do século XIX, passaram a chamar o regime político totalitário da Ditadura do Proletariado, combinado com o sistema econômico socialista, de comunismo. Em verdade o comunismo seria o objetivo final do marxismo, que foi essa concepção de via socialista para o atingimento do comunismo. Mas nunca se atingiu o comunismo. Só que os próprios socialistas passaram a chamar esse tipo de socialismo de estado, politicamente totalitário, de comunismo. Isso foi uma deturpação do significado da palavra comunismo, mas que acabou sendo absorvido.

por que a chuva cai em gotas ? não poderia cair tudo de uma vez ?‎

As nuvens se formam por condensação do vapor d'água atmosférico. Como ele é mil vezes mais volumoso do que a água líquida, a condensação promove a formação de gotículas de água micrométricas, separadas milimetricamente entre si. Isso é a nuvem, que está sempre em queda, evaporando na base e recondensando no topo. A chuva ocorre porque sementes de pó ou fumaça promovem a agregação de gotículas a elas, formando as gotas, que conseguem cair até o chão, sem que evaporem totalmente na queda. Quanto mais frio, menores as gotas. Se se estiver abaixo de zero grau, as gotinhas se congelam na queda, formando a neve. Já o granizo é congelado na própria nuvem. Não é possível cair a água toda de uma vez porque a formação da água provém de um vapor disperso em grande volume. Para todo o vapor formar uma massa contínua de água o volume teria que ser reduzido mil vezes e não há nada que promova essa redução. Então a redução e feita fracionadamente. Cada milimetro cúbico de vapor forma umas mil gotículas de nuvem de 10 micrometros de diâmetro.

É mais vantajoso mudar a si mesmo para cumprir melhor seu papel como cidadão (e ajudar a sociedade a "melhorar" a partir de si) ou tentar mudar a sociedade como um todo enquanto você mesmo continua "gerando problema"? Explique.

Depende. Se o desacordo entre o comportamento pessoal e o admissível pela sociedade se refere a algo que a pessoa pretenda fazer que seja, de fato, maléfico, então a pessoa deve mudar. Mas se se refere a algo benéfico, que a sociedade não admita por preconceito, então o que se precisa é mudar a sociedade.

Concorda com a chamada "amizade colorida"? Me fizeram algumas questões sobre esse tema e percebi que algumas pessoas se "espantam" com isso ainda. O que pensa sobre?

Uma amizade que envolva relacionamento sexual, para mim, é tão admissível quanto uma que não. Sem que isso enseje nenhum compromisso de permanência ou exclusividade. Trata-se de uma atividade perfeitamente normal como a curtição da companhia do amigo ou amiga para qualquer outro propósito. Se algum dos envolvidos já tiver outro relacionamento amoroso e sexual estável, vai depender de como esse relacionamento esteja estabelecido. Se ele admitir, explícita ou implicitamente, apenas a exclusividade, não será honesto quebrá-la. Mas ele pode admitir a não exclusividade, sendo que, nesse caso, é bom que seja explícito. Então não há mal nenhum. Um relacionamento sexual pode mesmo ocorrer entre pessoas, do mesmo ou de sexos opostos, que não mantenham uma relação de amizade. Há que se distinguir sexo, amor, relação e compromisso. Pode haver qualquer combinação desses fatos, o que resulta em 15 possibilidades.

Por que a hipótese de Avogrado não é chamada de teoria?‎

A chamada "Hipótese de Avogadro", epistemologicamente falando é uma lei, e não uma hipótese, pois é confirmada. A diferença entre Lei e Teoria é que a Lei (em ciência) é uma descrição empírica de um comportamento particular de algum fenômeno da natureza ou da sociedade. Uma Teoria, por outro lado, já é uma corporificação abrangente de explicações concernentes a fenômenos correlacionados. Uma teoria abrange várias leis, para cada fato que ela cogita. Além das leis a teoria incorpora definições e teoremas, que são conclusões derivadas das leis por raciocínio lógico. As leis não são deduzidas e sim induzidas. Também é possível que as teorias incorporem "Princípios", que não são induzidos observacional ou experimentalmente, mas são formulados de modo a que as leis possam deles serem deduzidas por raciocínio.

Qual é a causa da dualidade onda/partícula (luz)? É um evento sem causa assim como o decaimento radioativo? Ou não se tem conhecimento ainda?

A dualidade onda-partícula não é um evento, logo não possui causa. É uma característica do comportamento das quantizações de campo, quer materiais (férmions), quer radiantes (bósons). Ou seja, tais quantizações exibem tanto comportamento de partícula quanto de onda, dependendo do aspecto que esteja sendo analisado. Elas se propagam como ondas e interagem como partículas. Isso é um fato inerente à natureza. Não há que se buscar a causa desse comportamento. É o modo como a realidade se apresenta.

"[...]Geometricamente o argumento Kalam só se aplica se o eixo dos tempos for uma semi-reta, com seu ponto inicial infinitamente afastado. Mas não se aplica se o eixo dos tempos for uma reta".Já que é uma semi-reta, explique-me mais sobre isso.Tenho apenas 15 anos e muita curiosidade no assunto.

Se o eixo dos tempos for uma semi-reta, como de fato é, então ele não poderia ter uma origem infinitamente afastada para o passado. E, de fato, não tem. Isso está de acordo com o argumento Kalam. Mas, muitos consideram que o argumento Kalam impediria o tempo de ser uma reta, sem início e nem fim. Isso o argumento não prova que seja impossível. Ou seja, é possível que o tempo seja infinito para o passado, mesmo que ele, de fato, não seja. Muitos se baseiam no argumento Kalam para provar que o tempo teria que ter um início num passado finito. O argumento diz que ele não poderia ter um início infinitamente afastado para o passado, mas não diz que ele poderia não ter início nenhum. O fato de que ele tenha não resulta do argumento Kalam e sim de dados observacionais.

Uma teoria na ciência seria então algo que já está comprovado?‎

Sim. Caso ainda não seja comprovado é só uma hipótese. Mas o fato de estar comprovado não significa que seja definitiva, pois a ciência está sempre se corrigindo em razão de novas informações. A Teoria é considerada a explicação correta à vista dos conhecimentos disponíveis no momento. Mas sempre pode ser revista.

Qual o melhor país para um brasileiro migrar?

Depende de cada um. Depende do que ele considera que seja uma sociedade boa. Alguns podem adorar as ilhas dos mares do Sul. Outros a Finlândia ou a Islândia. Uns a Austrália, outros o Canadá. Ou ainda Israel, Arábia, Índia, Japão. Cada caso é um caso.

Por que o povo(geralmente) critica quem cuida do corpo ? é errado cultivar a boa aparência física ?‎

Não me consta que isso seja criticado. Claro que não é errado. Será errado só se a pessoa tiver uma fixação obsessiva pela aparência. Então já passa a ser uma patologia. Mas dentro dos limites de normalidade, cuidar do corpo e da aparência é algo muito bom.

É verdade que a terra parou por um dia? Se tivesse parado, que indícios que sobrariam que nós observaríamos deste '' fenômeno '' ?‎

Claro que não. Isso é impossível de acontecer. Só se ela sofrer uma violenta colisão com outro planeta ou se for parando (como está) lentamente. Mas, antes disso, o Sol já explodiu e não haverá mais Terra.

Qual a diferença de uma teoria e uma hipótese científica?

A hipótese é uma proposta de explicação a ser verificada. A teoria já foi verificada e constatado que está correta.

Qual a idade certa para aprender um novo idioma?‎

Quanto mais cedo melhor. O ideal é começar aprendendo quando se aprende a falar, aprendendo mais de um idioma em paralelo. Por exemplo, a mãe só fala com a criança em português e o pai em árabe. Então ela aprenderá os dois. Demorará mais tempo para falar, mas já falará ambos.

A física entende a natureza ou descreve a natureza ?‎

Tanto descreve quanto busca entender e compreender. Descrever é dizer o que cada fenômeno seja. Entender é saber suas causas, circunstâncias, limitações, aplicações. Compreender é saber como aquilo se relaciona com o resto do conhecimento sobre o mundo. A Física busca resposta para tudo isso.

Ernesto, quando seus filhos eram bebês, você adotou alguma medida pouco usual que achasse influiria no desenvolvimento deles? Tipo, há quem diga que deixar o bebê somente no berço em uma única posição prejudique seu aprendizado, ou coisas do tipo. Isso procede?‎

Procede. Os bebês têm que ser expostos a estímulos variados, visuais, auditivos, táteis, olfativos, gustativos, térmicos, cinestésicos e assim por diante. Para tal é preciso que fiquem o maior tempo possível soltos no chão, sobre um colchão ou edredon. Com toda a liberdade de se virar, engatinhar, pegar coisas, morder e tudo o mais. Assim eu criei meus filhos. Outra coisa é que não se deve falar em tatibitate com eles. Deve-se falar normalmente, certamente num linguajar simples, para que eles aprenda as palavras do modo correto.

Você (com quase um século de idade), já testemunhou, ou foi alvo de suas investigações, alguma "cura milagrosa"? Qual sua opinião sobre os supostos milagres religiosos.‎

Não acho que tenha quase um século de idade, mas apenas 0,64 séculos. Nunca presenciei nem tive notícia de nenhuma "cura milagrosa". Todas as curas de que tive conhecimento foram resultantes da eficácia da medicina. Milagres não existem. O que pode haver são casos raros de curas. Mas todas naturais.

Tenho a leve impressão de que toda vez que demonstro sentimentos por alguém, essa pessoa me deixa. O segredo é não demonstrar. Sim ou Claro?‎

De modo nenhum. Todo sentimento tem que ser demonstrado mesmo. Se a pessoa te deixa não é por esse motivo. Talvez você se revele muito possessivo ou muito "colado" na pessoa. Tem que deixar a pessoa amada completamente livre e solta. Não se pode mendigar reciprocidade de sentimento e nem se exigir nada em troca. Se a pessoa quiser, ela corresponderá, se não, paciência.

Quais são os melhores livros ou manuais de física para você?

Para o Ensino Médio é a coleção do PSSC (Physical Science Study Committee), editado no Brasil pela Edart e já esgotado. Nem tem comparação com os atualmente disponíveis no mercado. Pode ser achado em sebos. São quatro volumes mais o Advanced Topics Suplement, que não foi traduzido. Para Física Geral do Ensino Superior é o Curso de Física de Berkeley, em cinco volumes. Os dois primeiros foram traduzidos pela Edgard Blücher e também estão esgotados os três últimos existem em espanhol pela Reverté. Ou então os originais americanos. Outra excelente é a coleção do Alonso & Finn, em três volumes, da Addison-Wesley. E, sem dúvida, o Feynman Lectures on Physics, que, felizmente, foi traduzido para o português pela editora Bookman, com o título LIções de Física de Feyman. Esse são livros em que, realmente, se aprende Física e não apenas se treina para fazer exames vestibulares ou o ENEM. Outro bom é o Física Conceitual, de Paul Hewitt, em português, pela editora Bookman.

Professor, como foi na primeira vez em que você quis entrar para a faculdade, no que diz respeito a preparação para o vestibular? Qual é a sua dica de estudos?

Quando eu entrei para a faculdade, em 1968, cada curso tinha o seu vestibular separado e as provas eram todas abertas apenas. No caso do meu curso, Matemática, só tinha prova de matemática, português e física. Eu não estudei especialmente para o vestibular, Apenas recordei, em uns três meses, a matéria do científico. Não existiam cursinhos. Estudei sozinho. Mas eu sempre fui um ótimo aluno na escola, de modo que eu já sabia quase tudo. Meu conselho é sempre estudar para saber e não para nenhum exame. Sabendo a matéria, o que cair, em qualquer exame, do tipo que for, a pessoa resolve. Inclusive eu nunca fui de resolver muitos exercícios. Me concentrava mais na teoria, nos conceitos, em seu entendimento, especialmente nas restrições de sua aplicabilidade. E, principalmente, em física e matemática, em saber deduzir fórmulas, inclusive inéditas. Isso é extremamente importante. Na solução de problemas, jamais resolva-os numericamente. Sempre com letras, substituindo os números só depois de achar a expressão do resultado. Tem gente que, em português, acha que gramática não é importante, basta treinar redação. Eu discordo. Acho que gramática é importante sim e sempre me dediquei a estudá-la.

Você votaria em um militar para a presidência da república?‎

Poderia votar, dependendo de quem fosse e se tivesse um compromisso democrático e o respaldo de um grupo democrático bem forte. Não acho que por ser militar seria necessariamente um golpista, um conservador, um reacionário, um direitista, um preconceituoso ou possuidor de alguma outra característica que me levaria a jamais votar nele.

A terra é o centro do universo?‎

O Universo não tem centro. Seja ele finito ou infinito. No caso de infinito, isso é óbvio. No caso de finito, qualquer ponto está, em média, igualmente afastado de todos os demais pontos, nenhum sendo, portanto, o centro. É como se fosse um ponto de uma esfera em relação ao resto da superfície da esfera. Todos são equivalentes.

VC costuma pensar que só pq vc acredita que não exista, não tenha evidências, ou de acordo com seus conceitos não exista, não significa que não possa existir algo q vc desacredite? Ou vc pensa que possa talvez existir algo que vc desacredita? Como funciona com vc?‎

Para mim, tudo o que não exiba evidências sensoriais de que existe, só pode ter sua existência admitida se isso for comprovado por meios indiretos. Qualquer coisa. Como, por exemplo, campo elétrico, átomos e muito mais. No caso de deuses e espíritos, nem só não são evidentes como também não há provas de que existam. Então devo supor que não existam, até que alguma prova se apresente. Não é preciso provar que não existem. A falta de evidências e provas da existência é suficiente para a suposição. Note que essa suposição não é garantia. Mas permite que se aja e raciocine como se não existisse. Isso se aplica, também, ao caso de extraterrestres. Em todos esses casos, pode ser que existam, mas, até que se verifique, há que se supor que não existam. Mas a possibilidade de existência de extraterrestres e maior do que a de existência de deuses e espíritos.

Pergunto, se o poder pode tudo, existe algum limite pra ele?

O poder não pode tudo. Ele é limitado, primeiramente, pelas leis da natureza, que não conseguem ser contrariadas (milagres e mágicas não existem). Em segundo lugar, por outro poder que se lhe oponha. E em terceiro lugar pela resistência do objeto sobre o qual ele pretenda se aplicar. Que pode ser o povo, no caso de poder político, religioso ou militar. Em geral, o poder exercido por pessoas só é capaz de se realizar porque quem o exerce encontra apoio por parte de algum grupo de outras pessoas que também se beneficiam dele. Uma pessoa isolada só tem o poder que a sua força, resistência, dedicação e inteligência pessoal lhe proporcionam. Inclusive porque, no caso dessa pessoa dispor de recursos econômicos para exercer o poder, esses recursos foram por ela adquiridos em razão de sua capacidade por conta de sua inteligência, dedicação, resistência e força (força física mesmo). Ou por essas qualidades em seus ancestrais que lhe deixaram a riqueza como herança.

Garotos de 15 anos tem um Q.I mais alto que Albert, mas por quê? Qual o motivo? A nova geração e mais desenvolvida que a anterior?

O fato de uma pessoa ter um QI muito alto não significa que a atual geração tenha QI mais alto. É preciso fazer um levantamento estatístico. A razão, nesse caso, é genética. Mas isso também não significa que seus pais ou avós tenham sido muito inteligentes. Significa, apenas, que o acaso que lava à construção do DNA de cada um fez com que genes que privilegiem a inteligência, de muitas gerações anteriores dessa pessoa, fossem reunidos nela. Não tem nada de extraordinário nessa ocorrência. Quanto ao QI de Einstein, ele nunca foi medido, mas sim estimado, do mesmo modo que o de Newton ou Leonardo da Vinci. Além do mais, não é só o QI que é um indicador de inteligência.

Com a incessante quantidade de novas informações, como você seleciona o que quer aprender? Já que quer saber tudo sobre tudo.‎

Simples. Escolho o que me instiga, o que me fascina, o que não está bem estabelecido, o que é controverso, a fronteira do conhecimento. Isso é que é interessante. No caso, as neurociências, a nova cosmologia, o anarquismo, a economia de doações (não monetária), as lógicas alternativas, e assim vai. Não tenho interesse em ser um "expert" em nada. Gosto de saber de muitas coisas. Estudo pelo prazer que me dá. Não viso vantagem e nem busco a utilidade de nada que estudo. Só o conhecimento por si mesmo. E gosto de divulgá-lo, para abrir a mente das pessoas para possibilidades inusitadas. Mas não tanto que deixe o cérebro cair.

quinta-feira, 10 de abril de 2014

O que tem de novo na filosofia, ultimamente? Alguma ideia que valha ler? Ou só algumas ideias simples e não muito diferentes dos modelos propostos pelos filósofos mais antigos?

Há problemas de lógica em discussão. Há problemas metafísicos, como o da necessidade de causa, que ainda não estão completamente assentados. Há problemas epistemológicos. Há problemas éticos e morais. Há problemas em estética. Há muito assunto que é objeto de acaloradas discussões. Inclusive velhas questões na interface da filosofia com a religião, como as provas da existência de Deus. Isso, até hoje, é tema de debates, até pela internet. É preciso dispender um bom tempo fuçando pela internet até descobrir algo que se considere interessante. Por exemplo: a imoralidade da poligamia. Ou a estética da música Funk. Ou as lógica multi-dimensionais e difusas. Ou a polialética. Em filosofia política, também há muita discussão sobre a dicotomia direita-esquerda ou outros esquemas muiti-dimensionais. Há uma enorme leque de propostas, inclusive inteiramente inéditas a serem apresentadas em muitas áreas. A questão é que a Academia, em geral, é refratária a essas propostas, uma vez que não se enquadram no que, geralmente, é estudado. Parece que o mais moderno que se discute é Nietzsche. Para mim isso é um total antolhamento de visão filosófica.

a massa é constante, mas a velocidade aumenta, logo a energia da esfera tbm aumenta, então eu só tenho que fazer um circuito com várias depressões, e todas as vezes que a bolinha sair de uma depressão eu retiro a energia (e final - e inicial), e ela vai para a próxima receber + energia da gravidade‎

Se você fizer isso, ao voltar ao patamar de cima pela segunda vez, a bola terá menos velocidade. E ela vai diminuindo a cada depressão, até a bola parar. Em verdade, você terá extraído a energia que forneceu para a bola adquirir a sua velocidade inicial. Se fizer uma pista circular, a cada vez que a bola voltar à posição inicial, terá que dar um empurrão nela para que ela volte à velocidade com que começou. Então não será um moto-perpétuo, pois isso seria um dispositivo do qual você sempre pudesse extrair energia sem nunca fornecer.

Os EUA podem ser considerados ''Atenas'' do século XXI?

Não. De modo nenhum. Em termos políticos, para mim, o modelo de organização são os países escandinavos. Em termos de ciência e filosofia, não há um país hegemônico. Eu diria que os Estados Unidos, a Inglaterra, a França, a Alemanha, a Rússia, o Japão, a Índia e a China são os que mais desenvolvem conhecimento científico e tecnológico. Em filosofia, fico só com os Estados Unidos, a Inglaterra, a França e a Alemanha. O resto é periférico.

Professor, o que acha desses religiosos fanáticos que ameaçam as pessoas sem fé com ida ao inferno por ex. Ou qualquer outra coisa que gere medo ?‎

Acho que são pessoas medrosas. Apavoradas com a possibilidade de irem para o inferno. Consideram que se forem coniventes com alguém que seja ateu, se não lhes fizerem oposição, estariam pecando e merecendo o inferno. Acham que Deus exige delas que combatam a descrença para merecerem a salvação. Inclusive que combatam as crenças divergentes da sua. Cada fanático de cada religião está plenamente convencido que só a sua religião é detentora da verdade e que, portanto, os seguidores das outras e o ateus e agnósticos já estão condenados à danação eterna. E que ser amigo deles é um grande pecado. Só podem se aproximar de alguém fora de sua crença para tentar convertê-los. Alguns, em outros tempos (e, mesmo hoje) até acham que seria o seu dever exterminar todos os descrentes de sua fé. Isso e terrível. Uma total falta de amor ao próximo, que, em geral, é um mandamento da maioria das religiões.

Seria um paradoxo? Se eu fosse clonado (geneticamente idêntico) e meu clone tivesse todas minhas lembranças e sensações da minha vida, eu seria ele, pois somos os mesmo fisicamente?‎

Não. Mesmo um clone é outra pessoa. Para começar um clone não é formado com a mesma idade de quem ele é clonado. Ele é formado como um óvulo que vai ser gestado e nascer. Então terá toda uma outra vida, mesmo que com a mesma genética. Como dois gêmeos univitelinos. Se se conseguisse formar um clone com a mesma idade do ser de que ele é clonado e se transferissem as memórias (o que nem se tem a menor ideia de como ser feito), a partir do momento em que as memórias fossem transferidas, cada um dos dois começaria uma vida independente e memórias distintas iriam sendo formadas a partir da vivências de cada um. Se fossem a mesma pessoa, tudo o que uma percebesse a outra perceberia e os conteúdos das memórias seriam sempre os mesmos. Isso não tem como acontecer. O que as duas vão ter, a princípio, além da igualdade dos corpos em todos os aspectos, seria a mesma personalidade, o mesmo temperamento. Mas cada uma delas seria uma pessoa distinta.

"Portanto o que passa pela depressão chegará em menos tempo." mesmo que as forças aplicadas nos dois sejam iguais? :D se sim, vou construir um motor perpétum com base nisso, obrigado,‎

Não vejo como disso possa sair um moto-perpétuo. Essas bolas não se movem sob a ação de forças, pos estão com velocidade constante. Só na descida e na subida do buraco é que há uma resultante não nula. De qualquer modo, um moto-perpétuo e uma impossibilidade, a não ser que você não pretenda extrair trabalho nenhum dele e só ficar vendo ele funcionar. Isso sim, pode acontecer, como é o caso da rotação da Terra em torno de si e de sua revolução em torno do Sol. São perpétuas, mas não realizam trabalho sobre nada.

Mas se você se considera cético, como pode acreditar nas pessoas com facilidade ? uma vez que o ceticismo lhe coloca uma linha de pensamento, que desconfia de grande parte das coisas que advém de pessoas também ?

O ceticismo não se refere à descrença na sinceridade das pessoas no que elas dizem e sim na veracidade do que dizem. As pessoas podem não estar mentindo e nem falando a verdade. Porque elas acham que estão certas e estão equivocadas. O que eu confio é na sinceridade das pessoas, isto é, que elas não mintam em relação ao que consideram que seja a verdade. Mas desconfio de que elas possam não estar certas quanto à sua concepção do que seja a verdade. Percebeu a diferença?

E pra quem não tem interesse em ter um diploma de nível superior e vai seguir outro tipo de carreira da qual não precisa saber essas coisas? se torna totalmente inútil?‎

Nesse caso há duas alternativas. Ou a pessoa não faz o Ensino Médio Regular Geral e sim um profissionalizante da área em que vai querer atuar. Ou aprende essas coisas sem precisar mesmo, só para sua cultura pessoal. O fato de ser inútil não é ruim. As inutilidades podem ser prazerosas. Saber matemática, mesmo sem ter em que aplicar é uma extrema satisfação. Conhecer ciências é um grande prazer por poder saber como o mundo funciona. Saber história e geografia, então, é delicioso. E saber bem a língua portuguesa é ótimo para entender tudo o que se lê e tirar mais proveito. Em último caso, se não tiver interesse mesmo, não faça o Ensino Médio e vá trabalhar direto no que não precisa desses conhecimentos. Economiza-se tempo da vida.

http://photo4.ask.fm/308/831/936/710003036-1rb8gr2-alp9lm34c5htagk/preview/fsicaclassica.png qual chega primeiro ao lado direito? (mesma força, mesma massa, mesmo raio...)‎

Você quer dizer mesma velocidade inicial. Vamos considerar sem atrito.
O que não passa pela depressão leva um tempo que é a distância dividida pela velocidade média, que é constante.
O que passa pela depressão, ao descer aumenta a velocidade e move-se em baixo mais depressa. Depois diminui de novo, na subida, até o valor do começo e vai ao fim. Mas a velocidade média será maior do que no primeiro caso, pois houve um trecho de maior rapidez.
Portanto o que passa pela depressão chegará em menos tempo.

você nao acha q muitas matérias ensinadas são inuteis pro dia-a-dia?‎

Sim, mas não é para o dia a dia que elas são estudadas. É para dar uma cultura vasta à pessoa e prepará-la para o exercício profissional e para a compreensão do mundo, da sociedade, da natureza. Quem for pretender possuir um diploma de nível superior, seja em que área for, tem que ser uma pessoa culta em todas as áreas. Não basta ser um especialista em sua área. Tem que ter uma visão geral do mundo. Isso é o que as diversas disciplinas da Educação Básica se propõem a fazer. Advogados têm que saber trigonometria, Engenheiros têm que saber genética e evolução, Médicos têm que saber história, português e geografia. Senão vão ser uns ignorantes. Claro que faz muita falta um aprendizado de temas úteis para a vida cotidiana. Isso pode muito bem ser feito em um ensino em horário integral em que o aluno aprenderia, também, horticultura, marcenaria, culinária, corte e costura, mecânica de autos, eletrotécnica, hidráulica, construção civil, direção de veículos, digitação, contabilidade e muitas outras.

No século XVIII os filósofos consideravam todo o conhecimento humano, inclusive a ciência, que, em seu campo questionava muito sobre os porquês do Universo, só que, desde o século XIX isso tem mudado, os filósofos não estão sendo capazes de acompanhar o avanço das teorias cientificas[...]

Achei. Pois é. Isso é culpa dos filósofos, que não se dispõem a estudar ciências, especialmente física e biologia. No meu entendimento, inclusive, Filosofia deveria ser uma pós-graduação para quem fizesse uma graduação em Física e Biologia. Ou o curso de Filosofia deveria ter uns oito anos de duração e se estudar, também, Física e Biologia. Não precisa dedicar tempo aos aspectos técnicos dessas áreas, mas é preciso se entender de Mecânica Clássica, Termodinâmica (especialmente estatística), Eletromagnetismo, Ótica, Física Quântica, Física Nuclear, Física da Matéria Condensada, Relatividade, Cosmologia, Citologia, Biologia Molecular, Genética, Evolução, Ecologia. Senão não se pode ser filósofo. É assim que eu vejo a situação. Filosofia, absolutamente, não é do ramo das Ciências Humanas. Ela abrange todos os ramos: Humanas, Exatas e Biológicas. Como estava escrito no Pórtico da Academia de Platão: "Não entre se não for geômetra".

Professor, ajude-me por favor: Quanto maior a frequência de uma onda, maior também a energia e a velocidade de propagação, menor o comprimento de onda e menor o índice de refração ? é assim mesmo ?‎

A velocidade de propagação pode ser função da frequência quando o meio é dispersivo. Mas a variação é pequena. O que determina a velocidade são as propriedades inerciais e elásticas do meio ou, no caso de ondas eletromagnéticas, a permissividade elétrica e a permeabilidade magnética. O comprimento de onda vale a velocidade (de fase) dividida pela frequência. Mas é uma grandeza secundária. Quanto à energia, ela está ligada à amplitude da onda, e não à frequência. No caso de quantização de ondas eletromagnéticas é que cada quantum (o fóton) tem uma energia proporcional à frequência. Mas isso não acontece na onda considerada contínua.

[...] a ciência tornou-se demasiada técnica e matemática para os filósofos? Wittgenstein declarou certa vez: “A única tarefa que resta à filosofia é a análise da linguagem”. Qual é a sua opinião sobre?‎

Não acho não. Acho que os filósofos devem se dedicar a entender a ciência, especialmente Física e Biologia. E, é claro, Matemática. É só meter a cara e estudar. Os antigos filósofos também eram cientistas. Há muito na Filosofia que depende de conhecimento científico. Inclusive, para isso, estou escrevendo um livro "Física para Filósofos". Não é desculpa um filósofo dizer que não entende de Física. Estude e entenda, ora bolas. Do mesmo modo que cientistas têm que entender de Filosofia. Têm mesmo. E há muita coisa na Filosofia que requer conhecimento científico, como a Epistemologia, a Psicologia Filosófica, a Metafísica, a Lógica. Tudo isso tem a ver com o comportamento da natureza. Filósofos têm que entender, e não superficialmente, de ciências. Senão não conseguem apreender a realidade. Um curso de Filosofia teria que levar uns oito anos de estudo de graduação. É mais complexo do que Medicina. Quem não for, realmente, bem inteligente e muito dedicado não consegue fazer não.

Como diferir um sujeito 'entendido de filosofia', de um verdadeiro filósofo?

Quem é só entendido em filosofia apenas cita os outros filósofos mas não propõe nada de original. Nada que ele mesmo elaborou a partir de seus estudos e, principalmente, de suas reflexões. Os grandes filósofos foram grandes porque se rebelaram contra o que estava estabelecido e fizeram suas próprias propostas, em discordância. Os que apenas seguiram o que os outros disseram não são nem lembrados.

Pode acontecer do universo acabar e nunca mais haver vida ?

Mesmo que o Universo não acabe, as condições para a existência de vida, em qualquer lugar, não se manterão indefinidamente. Com a expansão o Universo vai ficando cada vez mais frio. Depois que todas as estrelas se apagarem, não existirá nenhuma condição para a vida em lugar nenhum. Na Terra isso acontecerá muito antes, mas porque o Sol vai se dilatar e aquecer tanto a Terra que acabará com toda a água.

O que causou mais dano a humanidade; o nazifacismo, a religião ou os brancos?‎

As religiões. Porque grande parte do mal que os brancos fizeram foi em nome delas. O próprio nazismo perseguiu os judeus em razão de um preconceito religioso. E as religiões promoveram muita guerra, muita matança de inocentes na inquisição. Também fizeram algo de bom. Mas tudo de bom que fizeram (caridade, balizamento moral) não requer religião para ser feito. Em geral os sentimentos nacionalistas, que se opõem à confraternização dos povos, tem fortes raízes religiosas.

Nunca evoluimos tanto na história da humanidade como nos ultimos 100 anos? Mudamos tudo nesse periodo,em ciência,tecnologia,meio de transporte,meios de comunicação e modo de pensar. É um privilégio nascer nessa era?‎

Não. O privilégio vai ser nascer depois que a humanidade já tiver resolvido todos os seus problemas de fome, miséria, ignorância, exploração, crimes, doenças. Depois que todo o mundo for um lugar pacífico, fraterno, harmônico, justo, aprazível e feliz. Isso ainda vai demorar vários séculos ou milênios. Aí é que será um privilégio existir.

A vida humana foi se aperfeiçoando com o tempo? no inicio não éramos tão inteligentes como nós somos hoje?

A mesma inteligência que temos hoje, já tínhamos desde que surgimos como espécie distinta, há 200 mil anos. A diferença é que hoje temos mais conhecimentos. A evolução do homem para uma nova espécie mais inteligente ainda não teve tempo de se concretizar. Mas acontecerá dentro de alguns milhões de anos.

LinkWithin

Related Posts with Thumbnails